Nutrition

Réussis tes devoirs et examens dès maintenant avec Quizwiz!

A postoperative client has been placed on a clear liquid diet. The nurse should provide the client with which items that are allowed to be consumed on this diet? Select all that apply 1. Broth 2. Coffee 3. Gelatin 4. Pudding 5. Vegetable juice 6. Pureed vegetables

1, 2, 3

The nurse instructs a client with chronic kidney disease who is receiving hemodialysis about dietary modifications. the nurse determines that the client understands these dietary modifications if the client selects which items from the dietary menu? 1. Cream of wheat, blueberries, coffee 2. Sausage and eggs, banana, orange juice 3. Bacon, cantaloupe melon, tomato juice 4. Cured pork, grits, strawberries, OJ

1. The diet for a client with chronic kidney disease who is receiving hemodialysis should include controlled amounts of sodium, phosphorus, calcium, potassium and fluids.

A client with parenteral nutrition (PN) infusing has disconnected the tubing from the central line catheter. The nurse assesses the client and suspects an air embolism. The nurse should immediately place the client in which position? 1. On the left side, with the head lower than the feet 2. On the left side, with the head higher than the feet 3. On the right side, with the head lower than the feet 4. On the right side, with the head higher than the feet

1. Rationale- Air embolism occurs when air enters the catheter system, such as when the system is opened for intravenous (IV) tubing changes or when the IV tubing disconnects. Air embolism is a critical situation; if it is suspected, the client should be placed in a left side-lying position. The head should be lower than the feet. This position is used to minimize the effect of the air traveling as a bolus to the lungs by trapping it in the right side of the heart. The positions in the remaining options are inappropriate if an air embolism is suspected.

The nurse is changing the central line dressing of a client receiving parenteral nutrition (PN) and notes that the catheter insertion site appears reddened. The nurse should next assess which item? 1. Client's temperature 2. Expiration of the bag 3. Time of last dressing change 4. Tightness of tubing connections

1. Rationale- Redness at the catheter insertion site is a possible indication of infection. The nurse would next assess for other signs of infection. Of the options given, the temperature is the next item to assess. The tightness of tubing connection should be assessed each time the PN is checked; loose connections would result in leakage, not skin redness. The expiration date on the bag is a viable option, but this also should be checked at the time the solution is hung and with each shift change. The time of the last dressing change should be checked with each shift change.

The nurse is caring for a client with cirrhosis of the liver. To minimize the effects of the disorder, the nurse teaches the client about foods that are high in thiamine. The nurse determines that the client has the best understanding of the dietary measures to follow if the client states an intention to increase the intake of which food? 1. Pork 2. Milk 3. Chicken 4. Broccoli

1. The client with cirrhosis needs to consume foods high in thiamine. Thiamine is present in a variety of foods of plant and animal origin. Pork products are especially rich in this vitamin.

A client receiving abdominal surgery has a large abdominal wound. The nurse should encourage the client to eat which food item that is naturally high in vitamin C to promote wound healing? 1. Milk 2. Oranges 3. Bananas 4. Chicken

2 Citrus fruits and juices are especially high in vitamin C

The nurse is conducting a dietary assessment on a client who is on a vegan diet. The nurse provides dietary teaching and should focus on foods high in which vitamin that may be lacking in a vegan diet? 1. Vitamin A 2. Vitamin B12 3. Vitamin C 4. Vitamin E

2. Vitamin B12: Vegans do not consume any animal products. Vitamin B12 is found in animal products.

The nurse is preparing to hang fat emulsion (lipids) and notes that fat globules are visible at the top of the solution. The nurse should take which action? 1. Rolls the bottle of solution gently 2. Obtains a different bottle of solution 3. Shakes the bottle of solution vigorously 4. Runs the bottle of solution under warm water.

2. Rationale- Fat emulsion (lipids) is a white, opaque solution administered intravenously during parenteral nutrition therapy to prevent fatty acid deficiency. The nurse should examine the bottle of fat emulsion for separation of emulsion into layers of fat globules or for the accumulation of froth. The nurse should not hang a fat emulsion if any of these are observed and should return the solution to the pharmacy. Therefore the remaining options are inappropriate actions.

The nurse is caring for a group of adult clients on an acute care medical-surgical nursing unit. The nurse understands that which client would be the least likely candidate for parenteral nutrition (PN)? 1. A 66 year-old client with extensive burns 2. A 42 year-old client who has had an open cholecystectomy 3. A 27 year-old client with severe exacerbation of Crohn's disease 4. A 35 year-old client with persistant nausea and vomiting from chemotherapy

2. Rationale- Parenteral nutrition is indicated in clients whose gastrointestinal tracts are not functional or must be rested, cannot take in a diet enterally for extended periods, or have increased metabolic need. Examples of these conditions include those clients with burns, exacerbation of Chron's disease, and persistent nausea and vomiting due to chemotherapy. Other clients would be those who have had extensive surgery, have multiple fractures, are septic, or have advanced cancer or acquired immunodeficiency syndrome. The client with the open cholecystectomy is not a candidate because this client would resume a regular diet within a few days following surgery.

The nurse is making initial rounds at the beginning of the shift and notes that the parenteral nutrition (PN) bag of an assigned client is empty. Which solution readily available on the nursing unit should the nurse hang until another PN solution is mixed and delivered to the nursing unit? 1. 5% dextrose in water 2. 10% dextrose in water 3. 5% dextrose in Ringer's lactate 4. 5% dextrose in 0.9% sodium chloride

2. Rationale- The client is at risk for hypoglycemia; therefore the solution containing the highest amount of glucose should be hung until the new PN solution becomes available. Because PN solutions contain high glucose concentrations, the 10% dextrose in water solution is the best of the choices presented. The solution selected should be one that minimizes the risk of hypoglcemia. The remaining options will not be as effective in minimizing the risk of hypoglycemia.

A client is being weaned from parenteral nutrition (PN) and is expected to begin taking solid food today. The ongoing solution rate has been 100 ml/hour. The nurse anticipated that which prescription regarding PN will accompany the diet prescription? 1. Discontinue the PN 2. Decrease PN rate to 50 mL/hour 3. Start 0.9% normal saline at 25 mL/hour 4. Continue current infusion rate prescription for PN

2. Rationale- When a client begins eating a regular diet after a period of receiving PN, the PN is decreased gradually. PN that is discontinued abruptly can cause hypoglycemia. Clients often have anorexia after beng without food for some time, and the digestive tract also is not used to producing the digestive enzymes that will be needed. Gradually decreasing the infusion rate allows the client to remain adequately nourished during the transition to a normal diet and prevents the occurrence of hypoglycemia. Even before clients are started on a solid diet, they are given clear liquids followed by full liquids to further ease the transition. A solution of normal saline does not provide the glucose needed during the transition of discontinuing the PN and could cause the client to experience hypoglycemia.

Which nursing action is essential prior to initiating a new prescription for 500 mL of fat emulsion (lipids) to infuse at 50 mL/hour? 1. Ensure that the client does not have diabetes 2. Determine whether the client has an allergy to eggs 3. Add regular insulin to the fat emulsion, using aseptic technique 4. Contact the health care provider (HCP) to have a central line inserted for fat emulsion infusion

2. Rationale-The client beginning infusions of fat emulsions must be first assessed for known allergies to eggs to prevent anaphylaxis. Egg yolk is a component of the solution and provides emulsification. The remaining options are unnecessary and are not related to the administration of fat emulsion.

A client has been discharged to home on parenteral nutrition (PN). With each visit, the home care nurse should assess which parameter most closely in monitoring this therapy? 1. Pulse and weight 2. Temperature and weight 3. Pulse and blood pressure 4. Temperature and blood pressure

2. Rationale-The client receiving PN at home should have her or his temperature monitored as a means of detecting infection, which is a potential complication of this therapy. An infection also could result in sepsis because the catheter is in a blood vessel. The client's weight is monitored as a measure of the effectiveness of this nutritional therapy and to detect hypervolemia. The pulse and blood pressure are important parameters to assess, but they do not relate specifically to the effects of PN.

A client who is recovering from a surgery has been advanced from a clear liquid diet to a full liquid diet. The client is looking forward to the diet change because he has been bored with the clear liquid diet. The nurse should offer which full liquid item to the client? 1. Tea 2. Gelatin 3. Custard 4. Ice pop

3

The nurse is planning to teach a client with malabsorption syndrome about the necessity of following a low-fat diet. The nurse develops a list of high-fat foods to avoid and should include which food item on the list? 1. Oranges 2. Broccoli 3. Cream Cheese 4. Broiled haddock

3 Cream cheese is a high fat food

A client receiving parenteral nutrition (PN) complains of a headache. The nurse notes that the client has an increased blood pressure, bounding pulse, jugular vein distention, and crackles bilaterally. The nurse determines that the client is experiencing which complications of PN therapy? 1. Sepsis 2. Air embolism 3. Hypervolemia 4. Hyperglycemia

3. Rationale- Hypervolemia is a critical situation and occurs from excessive fluid administration or administration of fluid too rapidly. Clients with cardiac, renal, or hepatic dysfunction are also at increased risk. The client's signs and symptoms presented in the question are consistent with hypervolemia. The increased intravascular volume increases the blood pressure, whereas the pulse rate increases as the heart tries to pump the extra fluid volume. The increased volume also causes neck vein distention and shifting of fluid into the alveoli, resulting in lung crackles. The signs and symptoms presented in the question do not indicate sepsis, air embolism, or hyperglycemia.

The nurse is preparing to hang the first bag of parenteral nutrition (PN) solution via the central line of an assigned client. The nurse should obtain which most essential piece of equipment before hanging the solution? 1. Urine test strips 2. Blood glucose meter 3. Electronic infusion pump 4. Noninvasive blood pressure monitor

3. Rationale- The nurse obtains an electronic infusion pump before hanging a PN solution. Because of the high glucose content, use of an infusion pump is necessary to ensure that the solution does not infuse too rapidly or fall behind. Because the client's blood glucose level is monitored every 4 to 6 hours during administration of PN, a blood glucose meter also will be needed, but this is not the most essential item needed before hanging the solution. Urine test strips (to measure glucose) rarely are used because of the advent of blood glucose monitoring. Although the blood pressure will be monitored, a noninvasive blood pressure monitor is not the most essential piece of equipment needed for this procedure.

The nurse monitoring the status of a client's fat emulsion (lipid) infusion and notes that the infusion is 1 hour behind. Which action should the nurse take? 1. Adjust the infusion rate to catch up over the next hour 2. Increase the infusion rate to catch up over the next two hours 3. Ensure that the fat emulsion infusion rate is infusing at the prescribed rate 4. Adjust the infusion rate to run wide open until the solution is back on time

3. Rationale- The nurse should not increase the rate of a fat emulsion to make up the difference if the infusion timing falls behind. Doing so could place the client at risk for fat overload. In addition, increasing the rate suddenly can cause fluid overload. The same principle (not increasing the rate) applies to PN or any intravenous (IV) infusion. Therefore the remaining options are incorrect.

The nurse is caring for a restless client who is beginning nutritional therapy with parenteral nutrition (PN). The nurse should plan to ensure that which action is taken to prevent the client from sustaining injury? 1. Calculate daily intake and output 2. Monitor the temperature once daily 3. Secure all connections in the PN system 4. Monitor blood glucose levels every 12 hours

3. Rationale- The nurse should plan to secure all connection tubing (tape is used per agency protocol). This helps prevent the restless client from pulling the connections accidentally. The nurse should also monitor intake and output, but this does not relate specifically to a risk for injury presented in the question. In addition, the client's temperature and blood glucose levels are monitored more frequently that the time frames identified in the options to detect signs of infection and hyperglycemia, respectively.

A client receiving parenteral nutrition (PN) suddenly develops a fever. The nurse notifies the health care provider (HCP) and the HCP initially prescribes that the solution and tubing be changed. What should the nurse do with the discontinued materials? 1. Discard them in the unit trash 2. Return them to the hospital pharmacy 3. Send them to the laboratory for culture 4. Save them for return to the manufacturer

3. Rationale- When the client who is receiving PN develops a fever, a catheter-related infection should be suspected. The solution and tubing should be changed, and the discontinued materials should be cultured for infectious organisms. The other options are incorrect. Because culture for infectious organisms is necessary, the discontinued materials are not discarded or returned to the pharmacy or manufacturer.

A client with hypertension has been told to maintain a diet low in sodium. The nurse who is teaching thi client about foods that are allowed should include which food item in a list provided to the client? 1. Tomato soup 2. Boiled shrimp 3. Instant oatmeal 4. Summer squash

4 Summer squash: Foods that are low in sodium include fruits and vegetables

The nurse is teaching a client who has iron deficiency anemia about foods she should include in her diet. The nurse determines that the client understands the dietary modifications if she selects which items from her menu? 1. Nuts and ilk 2. Coffee and tea 3. Cooked rolled oats and fish 4. Oranges and dark green leafy vegetables

4. Dark green leafy vegetables are a good source of iron nad oranges are a good source of vitamin C, which enhances iron absorption

A client receiving parenteral nutrition (PN) in the home setting has a weight gain of 5 lb in 1 week. The nurse should next assess the client for the presence of which condition? 1. Thirst 2. Polyuria 3. Decreased blood pressure 4. Crackles on auscultation of the lungs

4. Rationale- Optimal weight gain when the client is receiving PN is 1 to 2lb/week. The client who has a weight gain of 5lb/week while receiving PN is likely to have fluid retention. This can result in hypervolemia. Signs of hypervolemia include increased blood pressure, crackles on lung auscultation, a bounding pulse, jugular vein distention, headache, and weight gain more than desired. Thirst and polyuria are associated with hyperglycemia. A decreased blood pressure is likely to be noted in deficient fluid volume.

The nurse is preparing to change the parenteral nutrition (PN) solution bag and tubing. The client's central venous line is located in the right subclavian vein. The nurse asks the client to take which essential action during the tubing change? 1. Breath normally 2. Turn the head to the right 3. Exhale slowly and evenly 4. Take a deep breath, hold it, and bear down.

4. Rationale- The client should be asked to perform the Valsalva maneuver during tubing changes. This helps avoid air embolism during tubing changes. The nurse asks the client to take a deep breath, hold it, and bear down. If the intravenous line is on the right, the client turns is or her head to the left. This position increases intrathoracic pressure. Breathing normally and exhaling slowly and evenly are inappropriate and could enhance the potential for an air embolism during the tubing change.

A client is receiving parenteral nutrition (PN). The nurse monitors the client for complications of the therapy and should assess the client for which manifestations of hyperglycemia? 1. Fever, weak pulse, and thirst 2. Nausea, vomiting, and oliguria 3. Sweating, chills, and abdominal pain 4. Weakness, thirst, and increased urine output

4. Rationale-The high glucose concentration in PN places the client at risk for hyperglycemia. Signs of hyperglycemia include excessive thirst, fatigue, restlessness, confusion, weakness, Kussmaul's respirations, diuresis, and coma when hyperglycemia is severe. If the client has these symptoms, the blood glucose level should be checked immediately. The remaining options do not identify signs specific to hyperglycemia.

The nurse is admitting a 24 year old American American female client with a dx of rule-out anemia. The client has a hx of gastric bypass surgery for obesity 4 years ago. Current assessment findings include height 5'5, wt. 75 kg, P 110, R 27, and BP 104/66; pale mucous membranes and dyspnea on exertion. Which type of anemia would the nurse suspect the client has developed? A.Vitamin B12 deficiency B.Folic acid deficiency C.Iron deficiency D.Sickle cell anemia

A

The nurse writes a client problem of "activity intolerance" for a client dx with anemia. Which intervention should the nurse implement? A.Pace activities according to tolerance B.Provide supplements high in iron and vitamins C.Administer packed red blood cells D.Monitor vital signs q4h

A

The nurse is reviewing data collected from an adolescent patient suspected of having anorexia nervosa. Which findings should the nurse identify as contributing to this diagnosis? Standard Text: Select all that apply. A. distorted body image B. loss of control over food intake C. purging D. binge eating E. normal or above average body weight

A,B Anorexia nervosa is an eating disorder characterized by distorted body image. Anorexia nervosa is an eating disorder characterized by loss of control over food intake.

A patient is suspected of having protein calorie malnutrition (PCM) with a body mass index of less than 18. Which laboratory tests should the nurse expect to be prescribed for this patient? Standard Text: Select all that apply. A. serum albumin B. lymphocyte count C. serum electrolytes D. complete blood count (CBC) E. Urinalysis

A,B,C In PCM, serum albumin level reduced. In PCM, lymphocyte count is reduced. In PCM, serum electrolytes are measured. Potassium levels are low in severe malnutrition.

The goal of reducing or eliminating binge eating and purging behavior has been established for a patient with bulimia nervosa. What interventions should the nurse expect to be prescribed to help the patient achieve this goal? Standard Text: Select all that apply. A. nutritional counseling B. cognitive‒behavioral therapy C. antidepressants D. vitamin therapy E. hospitalization

A,B,C Nutritional counseling is directed at establishing a regular meal pattern and encouraging an appropriate amount of regular exercise. Cognitive‒behavioral therapy focuses on the patient's excessive concerns about weight, persistent dieting, and binge‒purge behaviors. Many times patients binge eat and then purge because they are depressed. Using antidepressants may help the bulimic patient to prevent a relapse.

The nurse is preparing information for a community seminar on the hazards of obesity. Which disorders should the nurse include as being complications of obesity? Standard Text: Select all that apply. A. cardiovascular diseases B. obstructive sleep apnea C. diabetes mellitus type 2 D. hypotension E. renal insufficiency

A,B,C Obesity leads to atherosclerosis, which increases vascular resistance, predisposing the patient to cardiovascular diseases. Respiratory airway collapse can occur during sleep in obese patients. Obesity increases the risk of developing diabetes mellitus type 2 in adults.

The nurse suspects that a patient has a vitamin C deficiency. What did the nurse assess to come to this conclusion? Standard Text: Select all that apply. A. delayed wound healing B. swollen bleeding gums C. depression D. night blindness E. muscle wasting

A,B,C Vitamin C is critical for wound healing. A manifestation of vitamin C deficiency is swollen, bleeding gums. A manifestation of vitamin C deficiency is depression.

The nurse suspects a patient is experiencing protein-calorie malnutrition. What did the nurse assess to come to this conclusion? (Select All that Apply) A. Thin Hair B. Dry flaking skin C. anxiety and agitation D. Recent 5lb weight loss E. Hyperactive bowel sounds

A,B,D Manifestations of protein-calorie malnutrition include thin hair, dry flaking skin, and a recent weight loss.

The nurse writes a dx of altered tissue perfusion for a client diagnosed with anemia. Which interventions should be included in the plan of care? Select all that apply. A.Monitor the clients hemoglobin and hematocrit B.Move the client to a room near the nurses desk C.Limit the clients dietary intake of green vegetables D.Assess the client for numbness and tingling E.Allow for rest periods during the day for the client

A,B,D,E

The home care nurse is planning care for a home-bound older adult who is losing an unplanned 1 to 2 pounds each month. What should this patients's plan of care include? (Select all that apply) A. Meals-On-Wheels deliveries B. Referral for diagnostic studies C. Use of nutritional supplements D. Follow-up by primary care physician E. Placement in residential care facility F. Transportation to congregate senior meals

A,C,D,F Many contributing factors influence nutrition in the home-bound older adult such as inability to shop for nutritional foods or the inability to afford and prepare nutritional food.

A patient has been researching medications to help achieve a weight loss goal. What is the medication classification that the nurse should review with the patient that could help meet the patient's goal? A. lipase inhibitor B. antiepileptic C. anticholinergics D. adrenergics

A. A lipase inhibitor reduces fat absorption from the GI tract.

A patient with a BMI of 29 says, "I cut out all my sweet snacks last week, and I still can't lose any weight." How should the nurse respond to this patient? A. "Let's calculate how many calories you are not eating each day." B. "I recommend that you go see a dietician." C. "I'll make a note in your file that you no longer eating sweet snacks." D. "You didn't gain the weight overnight."

A. A pound of body fat is equivalent to 3500 kilocalorie (kcal). To lose one pound, therefore, a person must reduce daily caloric intake by 250 kcal for 14 days or increase activity enough to burn the equivalent kcal.

The nurse is planning care for a patient with anorexia nervosa. Which problem should the nurse identify as a priority for this patient? A. inadequate oral intake B. feelings of adequacy C. loss of control D. skewed opinion of appearance

A. An inadequate oral intake negatively impacts all physiological processes. This is the priority for the patient at this time.

A patient who is 5 feet 5 inches tall and weighs 144 lbs asks the nurse if she would be considered obese. How should the nurse respond to this patient? A. "You are a normal weight for your height." B. "Yes, you are slightly obese for your height." C. "You are slightly overweight." D. "You are moderately obese."

A. BMI is calculated by dividing the weight (in kilograms) by the height in meters squared (m2). Utilizing the body mass index table, the patient has a body mass index of 24 kg/m2. A body mass index greater than 25 kg/m2 is considered overweight.

The mother of a teen is concerned that her daughter's nutritional status is compromised since the daughter has an increased interest in losing weight, weighs herself several times each day, and at times ingests large amounts of food. The daughter has not lost or gained much weight, but the mother wonders if her daughter has anorexia nervosa. How should the nurse respond to this mother? A. These are behaviors consistent with bulimia nervosa. B. These are behaviors consistent with early-onset anorexia nervosa. C. These are behaviors consistent with binge-eating disorder. D. These are behaviors consistent with a metabolic disorder.

A. Bulimia nervosa is a disorder in which patients eat large quantities of foods and then purge themselves by means of vomiting. Laxatives also may be employed.

The nurse is caring for an adolescent with anorexia nervosa. What should the nurse include in this patient's plan of care? A. Provide a variety of cold or room temperature foods B. Serve the patient three balanced meals per day. C. Discuss weight-gain needs with the patient. D. Observe the patient's activities for 15 minutes after eating.

A. Cold or room temperature foods are often more appealing to patients with anorexia nervosa.

An overweight patient states, "I'm trying to stick to my diet and exercise plan, but my spouse tells me that I'm fine the way I am." What type of problem is this patient experiencing? A. lack of family and social support to adhere to the plan B. eating more than is required for bodily functions C. difficulty with exercise and activity D. generalized feelings of self-reproach

A. Family and social support is critical to successful adherence to the therapeutic regimen. Without family support, the patient will have difficulty adhering to the weight loss plan.

During a physical assessment, the nurse suspects that the patient might be experiencing metabolic syndrome. Which assessment finding provides evidence for this nurse's assumption? A. blood pressure 150/96 B. difficulty ambulating C. low waist-to-hip ratio D. heart rate 72 and regular

A. Individuals with metabolic syndrome are found to have three or more specific manifestations, one of which is hypertension.

After following a structured diet, a patient diagnosed with diabetes mellitus is surprised to learn that his blood glucose levels have decreased and oral medications are no longer required. What explanation regarding the impact of diet on diabetes management should the nurse give the patient? A. Less body mass means less insulin is needed to maintain constant glucose levels. B. Body mass reduces cellular resistance to insulin. C. Reduced dietary intake of carbohydrates is responsible for the weight loss. D. Reduced dietary intake results in a reduced need for insulin.

A. Insulin is needed by the body cells to facilitate glucose transport across cell walls. The greater the body's mass, the increase in likelihood the body's cells will become resistant to insulin. This will result in type 2 diabetes.

The nurse is reviewing the lipid panel of a patient with a body mass index (BMI) of 31. What should the nurse expect this patient's values to be? A. low high-density lipoprotein (HDL) B. elevated HDL C. normal thyroid hormone (TH) level D. low-density lipoprotein (LDL)

A. LDL levels are elevated in obese patients.

An overweight patient tells the nurse, "Every Monday at work we have bagels. I can't stop myself! Sometimes I eat two!" What should the nurse realize this patient is describing? A. appetite stimulation by external cues B. extreme hunger from calorie restriction C. carbohydrate addiction in its early stage D. metabolic syndrome development

A. Most overweight people are stimulated to eat by external cues, such as the proximity to food and the time of day.

A patient who lives alone has a BMI of 34. What strategy should the nurse suggest to help this patient reduce overeating? A. Prepare a meal and eat it in the dining room. B. Eat out more often to control portion size. C. Read a book while eating as a distraction from the food. D. Cook once a week and store the leftovers to reduce the need to cook again.

A. One strategy to control the psychological response to food is to use attractive dinnerware, and prepare a formal setting for eating which would occur in a dining room.

The nurse hears a parent say to a child, "If you behave, we'll stop and get you an ice cream cone when we're done here." What should the nurse realize is occurring with the parent? A. rewarding behavior with food B. frustration with the child's behavior C. anxiety due to parenting D. hunger as a motivating factor

A. Sociocultural influences that contribute to obesity include overeating at family meals, rewarding behavior with food, religious and family gatherings that promote food intake, and sedentary lifestyles.

A patient planning to begin a weight loss diet asks the nurse for suggestions as to how to balance her eating. What information should the nurse provide to the patient? A. The diet should reduce calories to 1,000‒1,600 per day, with less than 10% of the total calories coming from fat. B. The diet should be between 750 and 1,000 calories per day, with less than 15% of the total calories coming from fat. C. The diet should simply cut 500 calories per day from the normal intake. D. The best diet will be between 1,250 and 1,500 calories per day, with 15% of the calories being sources of protein.

A. The best diet is a balance of all nutrients. Ideally, it should consist of 1,000‒1,600 calories per day and consist of no more than 10% fat.

The nurse teaches a patient about the medication orlistat (Xenical). Which patient statement indicates the need for additional teaching? A. "I should take this medication 30 minutes before eating." B. "This medication will reduce the amount of fat my body absorbs." C. "I will need to take supplements of vitamins A, D, E, and K daily." D. "A low-calorie diet will need to be followed."

A. The medication must be taken at mealtime or within the first hour of eating.

The client who is morbidly obese has undergone gastric bypass surgery. Which immediate postoperative intervention has the greatest priority? A. Monitor respiratory status. B. Weigh the client daily C. Teach a healthy diet. D. Assist in behavior modification.

A. The morbidly obese client will have a large abdomen, preventing the lungs from expanding, which predispose the client to respiratory complication.

While completing the health history, the nurse learns that a patient often eats excessive amounts of food when alone and when not hungry, and has intense feelings of self-disgust afterwards. The patient denies purging after these episodes. The nurse realizes that the patient is at risk of developing which health problem? A. type 2 diabetes mellitus B. type 1 diabetes mellitus C. dehydration D. electrolyte imbalances

A. The patient is demonstrating a binge-eating disorder. The excessive eating eventually will result in weight gain. Individuals with a body mass index greater than recommended are at an increased risk for the development of type 2 diabetes mellitus.

A patient regained 15 pounds that had been lost the previous year. What should the nurse suggest to this patient? A. Return to the diet, exercise, and behavior change techniques that worked before. B. Switch to a new diet in which the weight could be lost again in two weeks. C. Consider the possibility that the patient's body needs to have the extra 15 lbs. D. Understand that the increased weight does not make the patient obese.

A. The potential risks associated with regaining weight make maintenance a critical issue. Patients are encouraged to continue exercise, self-monitoring, and treatment support.

The female client presents to the clinic for an examination because she has not had a menstrual cycle several months and wonder if she could be pregnant. The client is 5'10" tall and weighs 45kg. Which assessment data should the nurse obtain first? A. Ask the client to recall what she ate for the last 24 hours. B. Determine what type of birth control the client has been using. C. Reweigh the client to confirm the data. D. Take the client's pulse and blood pressure.

A. This is client is 5'10" tall and weighs 99 pounds. Menses will cease if the client is severely emaciated. A 24-hour dietary recall is a step toward assessing the client's eating patterns.

A 45-year-old patient says, "I'm gaining weight but I'm not eating any differently than I did years ago." What should the nurse realize is occurring with this patient? A. gaining weight associated with aging B. denying the truth about overeating C. justifying the weight gain D. seeking approval to gain weight

A. To reduce weight gain commonly associated with aging, encourage patients to gradually reduce the amount of calories consumed.

A patient desiring to begin a very-low-calorie diet (VLCD) for rapid weight reduction is concerned about the safety of the diet. What information should the nurse provide to the patient? A. VLCDs are not recommended for people over age 50. B. VLCDs result in significant losses of muscle mass in response to the protein restriction. C. VLCDs are safe for patients who have a lower body mass index and need to lose a small amount of weight rapidly. D. VLCDs are safest for middle-aged and senior patients.

A. Very-low-calorie diets (VLCDs) are indicated for patients having elevated body mass indexes greater than 30 kg/m2. VLCD may not be appropriate for use in people over age 50 due to normal loss of lean body mass and adverse effects such as fatigue, constipation, nausea, diarrhea, and gallstone formation.

The nurse is preparing to administer total parenteral nutrition intravenously to a patient with malnutrition. What fat soluble vitamin should the nurse note is absent from the nutritional mixture? A. vitamin K B. vitamin A C. vitamin D D. vitamin E

A. Vitamin K is the only fat soluble vitamin that cannot be administered intravenously.

The nurse is helping a patient identify ways to adhere to a weight reduction plan. What should the nurse suggest to help this patient? A. Set aside small nonfood rewards when you meet a goal. B. Eat alone to reduce outside distractions. C. Drink water or a diet beverage after eating to promote feelings of fullness. D. Allow at least 45 minutes to 1 hour to promote full enjoyment of a meal.

A. When dieting, a small nonfood reward can serve as an incentive for working toward a goal.

A patient diagnosed with obesity asks about the appetite suppressant phentermine to assist with a weight loss program. Which information in the patient's health history might restrict the patient's ability to take this medication? A. Frequent use of alcohol B. History of narcolepsy C. A family history of thrombophlebitis D. A body mass index of 31kg/m2

A. Alcohol use or abuse can be contraindication for this medication.

Fats are composed of triglycerides and fatty acids. Triglycerides: A. Are made up of three fatty acids B. Can be saturated C. Can be monounsaturated D. Can be polyunsaturated

A. Are made up of three fatty acids Triglycerides circulate in the blood and are made up of three fatty acids attached to a glycerol. Fatty acids (not triglycerides) can be saturated or unsaturated (monounsaturated or polyunsaturated).

The energy needed to maintain life-sustaining activities for a specific period of time at rest is known as: A. BMR B. REE C. Nutrients D. Nutrient density

A. BMR The basal metabolic rate (BMR) is the energy needed to maintain life-sustaining activities for a specific period of time at rest. The resting energy expenditure (REE), or resting metabolic rate, is the amount of energy an individual needs to consume over a 24-hour period for the body to maintain all of its internal working activities while at rest. Nutrients are the elements necessary for body processes and function. Nutrient density is the proportion of essential nutrients to the number of kilocalories. High-nutrient density foods provide a large number of nutrients in relation to kilocalories.

Dietary reference intakes (DRIs) present evidence-based criteria for an acceptable range of amounts of vitamins and nutrients for each gender and age group. Components of DRIs include which of the following? (Select all that apply.) A. Estimated average requirement (EAR) B. Recommended dietary allowance (RDA) C. The Food Guide Pyramid D. Adequate intake (AI) E. The tolerable upper intake level (UL)

A. Estimated average requirement (EAR) B. Recommended dietary allowance (RDA) D. Adequate intake (AI) E. The tolerable upper intake level (UL) Dietary reference intakes (DRIs) present evidence-based criteria for an acceptable range of amounts of vitamins and nutrients for each gender and age group. DRIs have four components. The estimated average requirement (EAR) is the recommended amount of a nutrient that appears sufficient to maintain a specific body function for 50% of the population based on age and gender. The recommended dietary allowance (RDA) indicates the average needs of 98% of the population, not the exact needs of the individual. Adequate intake (AI) is the suggested intake for individuals based on observed or experimentally determined estimates of nutrient intakes and is used when evidence is insufficient to allow the RDA to be set. The tolerable upper intake level (UL) is the highest level that likely poses no risk of adverse health events. It is not a recommended level of intake. The food guide pyramid is not a component of the DRIs.

The patient is asking the nurse about the best way to stay healthy. The nurse explains to the patient that from a nutritional point of view, the patient should (Select all that apply.) A. Maintain body weight in a healthy range B. Increase physical activity C. Increase intake of meat and other high-protein foods D. Keep total fat intake to 10% or less E. Choose and prepare foods with little salt

A. Maintain body weight in a healthy range B. Increase physical activity E. Choose and prepare foods with little salt According to the 2005 Dietary Guidelines for Americans, key recommendations include maintaining body weight in a healthy range; increasing physical activity and decreasing sedentary activities; increasing intake of fruits, vegetables, whole grain products, and fat-free or low-fat milk with less red meat; keeping fat intake between 30% and 35% of total calories, with most fats coming from polyunsaturated or monounsaturated fatty acids (most meats contain saturated fatty acids); and choosing prepared foods with little salt while at the same time eating potassium-rich foods.

The nurse is providing home care for a patient diagnosed with AIDS. In preparing meals for this patient, the nurse should: A. Provide small, frequent nutrient-dense meals B. Encourage intake of fatty foods to increase caloric intake C. Prepare hot meals because they are more easily tolerated D. Avoid salty foods and limit liquids to preserve electrolytes

A. Provide small, frequent nutrient-dense meals Small, frequent, nutrient-dense meals that limit fatty foods and overly sweet foods are easier to tolerate. Patients benefit from eating cold foods and drier or saltier foods with fluid in between

In determining the nutritional status of a patient and developing a plan of care, it is important to evaluate the patient according to: A. Published standards B. Nursing professional standards C. Absence of family input D. Patient input only

A. Published standards Referring to professional standards for nutrition is especially important during this step because published standards are based on scientific findings. Nursing standards cannot be used alone. Other health care professionals must be consulted to adopt interventions that reflect the patient's needs. Family should be involved in evaluation and design of interventions. Although patient input is important, synthesis of patient information from multiple sources is necessary to devising an individualized approach to care that is relevant to the patient's needs.

When expected nutritional outcomes are not being met, the nurse should: A. Revise the nurse measures or expected outcomes B. Alter the outcomes based on nursing standards C. Ensure that patient expectations are congruent with the nurse's expectations D. Readjust the plan to exclude cultural beliefs

A. Revise the nurse measures or expected outcomes When expected outcomes are not met, the nurse should revise the nursing measures or expected outcomes based on the patient's needs or preferences, not solely on the basis of nursing standards. Expectations and health care values held by nurses frequently differ from those held by patients. Working closely with patients enables the nurse to redefine expectations that are realistically met within the limits of conditions and treatments and to identify their dietary preferences and cultural beliefs.

The patient has just started on enteral feedings but is complaining of abdominal cramping. The nurse should: A. Slow the rate of tube feeding B. Instill cold formula to "numb" the stomach C. Place the patient in a supine position D. Change the tube feeding to a high-fat formula

A. Slow the rate of tube feeding One possible cause of abdominal cramping is a rapid increase in rate or volume. Lowering the rate of delivery may increase tolerance. Another possible cause of abdominal cramping is use of cold formula. The nurse should warm the formula to room temperature. The nurse should maintain the head of the bed at least 30 degrees. High-fat formulas are also a cause of abdominal cramping.

To create a new nutritional plan of care for a patient, the nurse needs to do which of the following? (Select all that apply.) A. Utilize the characteristics of a normal nutritional status B. Evaluate previous patient responses to nursing interventions C. Exclude established expected outcomes to evaluate patient responses D. Design innovative interventions to meet the patient's needs E. Follow through with evaluation and counseling

A. Utilize the characteristics of a normal nutritional status B. Evaluate previous patient responses to nursing interventions D. Design innovative interventions to meet the patient's needs E. Follow through with evaluation and counseling To create a new nutritional plan of care, the nurse must utilize characteristics of a normal nutritional status to gauge effectiveness of the plan. The nurse must be aware of previous patient responses to nursing interventions for altered nutrition to determine the probability of success. The nurse must use established expected outcomes to evaluate the patient's response to care (e.g., patient's weight increases by 0.5 kg/week). The nurse must also be creative when designing innovative nursing interventions to meet the patient's nutritional needs and must demonstrate responsibility by following through with evaluation and counseling to successfully reach goals.

The nurse is concerned about pulmonary aspiration when providing her patient with tube feedings. The nurse should: A. Verify tube placement before feeding B. Lower the head of the bed to a supine position C. Add blue food coloring to the enteral formula D. Run the formula over 12 hours to decrease volume

A. Verify tube placement before feeding A major cause of pulmonary aspiration is regurgitation of formula. The nurse needs to verify tube placement and elevate the head of the bed 30 to 45 degrees during feedings and for 2 hours afterward. The addition of blue food coloring to enteral formula to assist with detection of aspirate is no longer used. Do not hang formula longer than 4 to 8 hours. Formula becomes a medium for bacterial growth after that length of time.

When developing a plan of care for a patient with altered nutritional needs, the nurse must assess the patient for which of the following? (Select all that apply.) A. What is the condition now? B. Is the condition stable? C. Will the condition get worse? D. Will the disease process accelerate deterioration? E. Which single objective measure will predict the course of action?

A. What is the condition now? B. Is the condition stable? C. Will the condition get worse? D. Will the disease process accelerate deterioration? Nutritional screening tools must gather data based on four main principles: What is the condition now? Is the condition stable? Will the condition get worse? And will the disease process accelerate nutritional deterioration? Using a single objective measure is ineffective in predicting risk of nutritional problems.

An RN who usually works on the pediatric unit is floated to the GI medical-surgical unit. Which client is appropriate for the charge nurse to assign to the float nurse? A. 20-year-old with anorexia nervosa receiving total parenteral nutrition (TPN) through a central venous line B. 35-year-old who had a laparoscopic gastroplasty yesterday and is now taking sips of clear liquids C. 60-year-old with gastric cancer receiving elemental feedings through a jejunostomy tube D. 65-year-old with morbid obesity who requires a preoperative bariatric surgery assessment

Answer A. Rationales: Correct: A pediatric nurse would be familiar with the pathophysiology and collaborative treatment of the client with anorexia nervosa. Incorrect: The client with a laparoscopic gastroplasty requires more familiarity with adult nutritional disorders and bariatric surgery. Incorrect: The client with gastric cancer receiving elemental feedings through a jejunostomy tube requires more familiarity with adult nutritional disorders and bariatric surgery. Incorrect: The client with morbid obesity who requires a preoperative bariatric surgery assessment requires more familiarity with adult nutritional disorders and bariatric surgery.

Situation: An 87-year-old woman resident from an extended-care facility has not been eating for several days. She is admitted to the hospital with a diagnosis of malnutrition. She has an enteral feeding tube placed in her left nostril. Her medications include digoxin (Lanoxin), ranitidine (Zantac), and potassium chloride elixir (Kay Ciel). She has developed a severe case of diarrhea. What is a possible cause? A. Digoxin (Lanoxin) B. Gastritis C. Potassium chloride (Kay Ciel) D. Ranitidine (Zantac)

Answer C. Rationales: Incorrect: Diarrhea is not a frequent side effect of digoxin. Incorrect: Gastritis does not cause diarrhea. The other signs and symptoms of gastritis are not mentioned in this scenario. Correct: In some cases, diarrhea may be the result of liquid medications such as elixirs and suspensions that have a very high osmolality. Incorrect: Diarrhea is not a frequent side effect of ranitidine.

A nurse is monitoring a client who is receiving an intravenous fat emulsion (IVFE) nutritional supplement. What action does the nurse take in the event that the client develops fever, increased triglycerides, and clotting problems? A. Discontinues the IVFE infusion B. Documents the findings and continues to monitor C. Slows the rate of flow of the IVFE infusion D. Switches the infusion to total parenteral nutrition (TPN) infusion

Answer A. Rationales: Correct: For clients receiving fat emulsions, monitor for manifestations of fat overload syndrome, especially in those who are critically ill. These manifestations include fever, increased triglycerides, clotting problems, and multi-system organ failure. Discontinue the IVFE infusion, and report any of these changes to the health care provider immediately if this complication is suspected. Incorrect: Documenting the findings and continuing to monitor will have serious repercussions for this client. The IV must be stopped. Incorrect: Slowing the rate of flow of the IVFE infusion will present a serious safety risk for the client. The IVFE needs to be stopped. Incorrect: Nurses do not request IV parenteral therapies or change them unless the health care provider makes the decision.

A client is receiving nutritional supplements to restore nutritional status. What does the nurse do to assess the effectiveness of the supplements for the client? A. Keeps an accurate and precise food and fluid intake record daily B. Makes certain the client is weighed daily at the same time C. Monitors vital signs every 4 hours and as needed D. Weekly assesses the client's skin for evidence(s) of breakdown

Answer B. Rationales: Incorrect: Although it is important to identify everything that the client is taking in orally, this does not help assess the effects of nutritional supplements on the client. Correct: Daily weigh-ins will best show the effects of nutritional supplements by showing how much weight the client is regaining. Incorrect: Although monitoring of vital signs is important, it does not help assess the effects of nutritional supplements on the client. Incorrect: Although it is important to identify any evidence of skin breakdown, this does not directly help in assessing the effects of nutritional supplements on the client.

A malnourished client is being discharged on enteral nutrition products. Which suggestion from the registered dietitian does the nurse implement to make the enteral feeding experience more normal for the client? A. Administering the feeding product on a regular schedule Incorrect B. Bringing the enteral product and napkin to the client on a tray C. Emphasizing the need to take iron medications before the feeding D. Once feeding is completed, putting equipment out of view

Answer B. Rationales: Incorrect: Although the feeding product should be administered according to the prescribed schedule, this will not necessarily normalize the experience for the client. Correct: "Serving" the enteral product and napkin on a tray will help normalize the feeding experience for the client. Incorrect: Although iron medications may be helpful in preventing constipation, encouraging their use will not normalize the experience for the client. Incorrect: Although putting equipment away after use may be helpful in taking the client out of the dependent "client" role, this will not serve to normalize the feeding experience itself.

A client has a primary problem of inadequate nutrition caused by the effects of chemotherapy. The client is receiving continuous enteral feedings through a nasogastric tube (NG) tube. What does the RN ask the LPN/LVN to do for this client? A. Assess nutritional parameters on the client every 3 days. B. Check the residual volume of the NG tube every 4 hours. C. Monitor the client for signs and symptoms of pneumonia. D. Teach the client about the purpose of enteral feedings.

Answer B. Rationales: Incorrect: Assessing nutritional parameters on the client is complex and requires broad knowledge about the physiology associated with malnutrition and possible complications of tube feedings. This activity should be performed by an RN. Correct: Checking the residual volume of the client's NG tube every 4 hours is within the scope of knowledge and practice for the LPN/LVN. Incorrect: Monitoring the client for signs and symptoms of pneumonia is complex and requires broad knowledge about the physiology associated with possible complications of tube feedings. This activity should be performed by an RN. Incorrect: Teaching the client about the purpose of enteral feedings is complex and requires broad knowledge about the physiology associated with malnutrition and possible complications of tube feedings. This activity should be performed by an RN.

A nurse is teaching a class of older adults in the community about engaging in "regular" exercise. What does the nurse advise them? A. "1 to 2 hours of cardiovascular exercise every day is a good idea." B. "Joining a fitness program or gym will help greatly with your exercise." C. "Walking 20 minutes provides the same benefit as long periods of exercise." D. "You will benefit most if you get into a group that shares your exercise goals."

Answer C. Rationales: Incorrect: 1 to 2 hours of cardiovascular exercise every day is not required to achieve benefits of exercise. Incorrect: A fitness program or gym is not necessary to achieve a regular exercise workout. It is expensive, and many older adults have a fixed income and cannot afford memberships. Correct: Although some people think that regular exercise has to include joining a fitness program or exercising for long periods of time, simple forms of exercise like walking 20 minutes provide the same type of benefit. Older adults can engage in this type of exercise. It does not cost money (like joining a program) and provides health benefits such as strengthening joints and improving cardiovascular health. Incorrect: A 20-minute walk can be accomplished with a group (such as "mall walking") or alone. Some people like and want to have this time to themselves.

A nurse manager in a long-term care facility plans nutritional assessments of all residents. Which nutritional assessment activity does the nurse delegate to unlicensed assistive personnel (UAP) at the facility? A. Assessing residents' abilities to swallow B. Determining residents' functional status C. Measuring the daily food and fluid intake of residents D. Screening a portion of the residents with the Mini Nutritional Assessme

Answer C. Rationales: Incorrect: Assessing residents' abilities to swallow requires broad knowledge of normal physiology, nutrition, and factors that impact on nutrition and should be done by licensed nursing staff. Incorrect: Determining residents' functional status requires broad knowledge of normal physiology, nutrition, and factors that impact on nutrition and should be done by licensed nursing staff. Correct: UAP education includes measurement of clients' oral intake; this skill does not require clinical judgment to be completed accurately. Incorrect: Screening with the Mini Nutritional Assessment requires broad knowledge of normal physiology, nutrition, and factors that impact on nutrition and should be done by licensed nursing staff.

A client receiving total parenteral nutrition (TPN) exhibits symptoms of congestive heart failure (CHF) and pulmonary edema. Which complication of TPN is the client most likely experiencing? A. Calcium imbalance B. Fluid volume deficit C. Fluid volume overload D. Potassium imbalance

Answer C. Rationales: Incorrect: This client's symptoms are not indicative of calcium imbalance. Incorrect: This client's symptoms are not indicative of fluid deficit. Correct: Congestive heart failure and pulmonary edema are symptoms of fluid overload Incorrect: This client's symptoms are not indicative of potassium imbalance.

Situation: An 87-year-old woman resident from an extended-care facility has not been eating for several days. She is admitted to the hospital with a diagnosis of malnutrition. She has an enteral feeding tube placed in her left nostril. Her medications include digoxin (Lanoxin), ranitidine (Zantac), and potassium chloride elixir (Kay Ciel). The nurse who is responsible for checking the gastric pH of the feeding tube tests it and obtains a value of 6.0. This finding may indicate that the feeding tube is in the client's lungs. Is there another possible explanation for the nurse to consider? A. No. The feeding tube must be removed. B. No. The potassium effect will prevent the pH from reaching 6.0. C. Yes. The client is taking Zantac. D. Yes. The pH paper has expired and is giving a false reading.

Answer C. Rationales: Incorrect: This finding-given the circumstances-does not mean that the tube is displaced and in the client's lungs. Incorrect: The potassium effect does not cause the pH to become more alkaline. Correct: The pH may be as high as 6.0 if the client takes certain medications, such as H2 blockers (e.g., ranitidine [Zantac], famotidine [Pepcid]). Incorrect: Expired pH paper will provide no data that are reliable, so it would be impossible to have a reading of "6."

A nurse is performing a health assessment on an obese client. The client states, "I have tried many diets in an effort to lose weight but have been unsuccessful!" How does the nurse assess whether the client's response to stress is related to the client's obesity? A. "Do you have a history of mental problems, especially depression?" B. "Do you usually use alcohol or drugs when you feel stressed?" C. "Tell me what you do to relieve stress in your daily life." D. "What is it about your obesity that causes you to feel uncomfortable?"

Answer C. Rationales: Incorrect: This question will cause the client to feel uncomfortable with the assessment. Problems in handling stress do not mean mental health or depression problems. Incorrect: This question could cause the client to feel uncomfortable with the assessment. More effective methods can be used to determine the client's alcohol and drug habits. Correct: This is the only question that allows the client to verbalize stress-relieving mechanisms. It is also a question that cannot be answered with a simple "yes" or "no." Incorrect: This question will only cause the client to restate the obvious. It does not determine the effect that stress has on the client.

A young adult man says that he cannot stay on a diet because of trouble finding one that will incorporate his food preferences. How does the nurse effectively plan nutritional care for this client? A. Calculates his body mass index (BMI) B. Keeps a 24-hour diary of his physical activities C. Maintains a 24-hour recall (diary) of his food intake D. Obtains his accurate height and weight measurements

Answer C. Rationales: Incorrect: Although calculating a BMI is an important part of a nutritional assessment, it does not address the issue of the client's food preferences. Incorrect: Keeping an activity diary will not reveal any information related to the client's food preferences. Correct: Maintaining a 24-hour recall of food intake will determine the client's food preferences and eating patterns so that they can be incorporated into the diet to the greatest extent possible. Incorrect: Although measuring height and weight is an important part of a nutritional assessment, it does not address the issue of the client's food preferences.

A client who is receiving total enteral nutrition (TEN) exhibits acute confusion and shallow breathing and says, "I feel weak." As the client begins to have a generalized seizure, how does the nurse interpret this client's signs and symptoms? A. The enteral tube is misplaced or dislodged. B. Abdominal distention is present. C. A fluid and electrolyte imbalance is present. D. This is refeeding syndrome.

Answer D. Rationale: Incorrect: If the enteral tube becomes misplaced or dislodged, the client may develop aspiration pneumonia displayed by increased temperature, increased pulse, dehydration, diminished breath sounds, and shortness of breath. Incorrect: Abdominal distention is most frequently accompanied by nausea and vomiting. Incorrect: Signs and symptoms of fluid and electrolyte problems resulting in circulatory overload can include peripheral edema, sudden weight gain, crackles, dyspnea, increased blood pressure, and bounding pulse. Correct: Symptoms of refeeding syndrome include shallow respirations, weakness, acute confusion, seizures, and increased bleeding tendency.

A nurse obtains assessment data on a client who had bariatric surgery today. Which finding does the nurse report to the surgeon immediately? A. Bowel sounds are not audible in all quadrants. B. Client's skin under the panniculus is excoriated. C. The client reports pain when being repositioned. D. Urine output total is 15 mL for the past 2 hours.

Answer D. Rationales: Incorrect: Inaudible bowel sounds may require nursing interventions but do not require immediate intervention by the surgeon. On the day of surgery, they will probably be absent normally for some time. Incorrect: Excoriated skin under the panniculus may require nursing interventions but does not require immediate intervention by the surgeon. Incorrect: Subjective reports of pain may require nursing interventions but do not require immediate intervention by the surgeon, as does the scant urine output. Correct: Oliguria may indicate severe postoperative complications such as anastomotic leaks or acute kidney failure.

Which client on a medical-surgical unit does the charge nurse assign to the LPN/LVN? A. 28-year-old with morbid obesity who had bariatric surgery today B. 30-year-old recently admitted with severe diarrhea and Clostridium difficile infection C. 36-year-old whose family needs instruction about how to use a gastric feeding tube D. 39-year-old with a jejunal feeding tube who needs elemental feedings administered

Answer D. Rationales: Incorrect: Initial assessment of a postoperative client requires RN education and scope of practice. Incorrect: Initial assessment of a new admission requires RN education and scope of practice. Incorrect: Client and family teaching requires RN education and scope of practice. Correct: LPN/LVN education includes administration of tube feedings and associated client care and monitoring.

An RN receives the change-of-shift report about these clients. Which client does the nurse assess first? A. 30-year-old admitted 2 hours ago with malnutrition that is associated with malabsorption syndrome B. 45-year-old who had gastric bypass surgery and is reporting severe incisional pain C. 50-year-old receiving total parenteral nutrition (TPN) with a blood glucose (BG) level of 300 mg/dL D. 75-year-old with dementia who is receiving nasogastric feedings and has a respiratory rate of 38 breaths/min

Answer D. Rationales: Incorrect: The client admitted 2 hours ago with malnutrition needs assessments and/or interventions by the RN, but maintaining respiratory function in the client with tachypnea is the highest priority. Incorrect: The client who had gastric bypass surgery and is reporting severe incisional pain needs assessments and/or interventions by the RN, but maintaining respiratory function in the client with tachypnea is the highest priority. Incorrect: The client receiving TPN with a BG level of 300 mg/dL needs assessments and/or interventions by the RN, but maintaining respiratory function in the client with tachypnea is the highest priority. Correct: Aspiration is a major complication in clients receiving tube feedings, especially in clients with an altered level of consciousness. This client needs respiratory assessment and interventions immediately.

A client is discharged home with an enteral feeding tube. What does the home health nurse do to determine the patency of the client's enteral tube? A. Arranges for the client to have an x-ray performed periodically B. Auscultates the client's abdomen for bowel sounds before each feeding C. Instills air into the tube to check for placement and patency before each feeding D. Tests aspirated tube contents for pH level before each feeding Correct

Answer D. Rationales: Incorrect: The client should have an x-ray performed when the enteral tube is initially inserted. Incorrect: The presence of bowel sounds does not indicate that the enteral tube is in place. Incorrect: This traditional auscultatory method for checking enteral tube placement is not reliable, especially for the client with a small-bore tube. Correct: This is considered to be the most accurate method for confirming enteral tube placement.

Which nursing care activity for a malnourished client does the nurse safely delegate to unlicensed assistive personnel (UAP)? A. Completing the Mini Nutritional Assessment B. Determining body mass index (BMI) C. Estimating body fat using skin-fold measurements D. Measuring current height and weight

Answer D. Rationales: Incorrect: The nurse is responsible for completing the Mini Nutritional Assessment. Incorrect: The nurse is responsible for determining the client's BMI. Incorrect: The nurse is responsible for estimating body fat using skin-fold measurements. Correct: Determining height and weight is the only activity that can be safely delegated to UAP.

An older adult client needs additional dietary protein but refuses to drink the prescribed liquid protein supplements. Which nursing intervention is most effective in increasing the client's protein intake? A. Administering the liquid supplement with routine medications Incorrect B. Giving a glucose polymer modular supplement C. Keeping a food and fluid intake diary for at least 3 days D. Providing protein modular supplements in the form of puddings

Answer D. Rationales: Incorrect: This approach will not be effective because the client has already refused to drink the liquid supplements. Incorrect: Glucose polymer modular supplements will increase the client's calorie intake but not protein intake. Incorrect: A food and fluid diary will provide information about the client's typical intake pattern but will not increase protein intake. Correct: Providing protein modular supplements in the form of puddings would increase the client's protein intake in an alternate format, other than a liquid supplement.

The head of the bed of a patient who is receiving enteral feedings is elevated to 45 degrees. Which complication associated with enteral feedings does this intervention help prevent? 1) Aspiration 2) Diarrhea 3) Infection 4) Electrolyte imbalance

Answer: 1) Aspiration Rationale: The head of the bed (HOB) should be elevated to at least 30 to 45 degrees during enteral feeding administration to prevent regurgitation and aspiration. Maintaining an elevated HOB aids in digestion by helping the transit of feeding through the GI tract. Diarrhea, infection, and electrolyte imbalance are all complications associated with enteral feeding, but cannot be prevented by elevating the head of the bed.

Patients may be deficient in which vitamin during the winter months? 1) A 2) D 3) E 4) K

Answer: 2) D Rationale: The body can synthesize vitamin D from a cholesterol compound in the skin when exposed to adequate sunlight. People at risk for vitamin D deficiency are those who spend little time outdoors; older people; and people who live in an institution (e.g., a nursing home). The deficiency can also occur in the winter at northern and southern latitudes, in people who keep their bodies covered (e.g., traditional Muslim women), and in those who use sunscreen. Also, because breast milk contains only small amounts of vitamin D, breastfed infants who are not exposed to enough sunlight are at risk of the deficiency and rickets. There is no seasonal tie to deficiencies in the other fat-soluble vitamins, A, E, and K.

Which action should the nurse take after administering a dose of medication through a percutaneous endoscopic gastrostomy (PEG) tube? 1) Continue the enteral feeding. 2) Flush the tube with 30 mL of water. 3) Wait 2 hours before resuming the feeding. 4) Check residual volume.

Answer: 2) Flush the tube with 30 mL of water. Rationale: The nurse should flush the PEG tube with 30 mL of water before and after administering a medication through the tube. The tube feeding should be held for 1 hour before and 1 hour after administering some medications, such as phenytoin (Dilantin). It is not necessary to hold the feeding for 2 hours. Residual volume should not be checked immediately after administering medications.

During parenteral nutrition administration, a nurse breaks sterile technique. For which complication does this place the patient at risk? 1) Air embolism 2) Sepsis 3) Thrombosis 4) Pneumothorax

Answer: 2) Sepsis Rationale: A break in sterile technique places the patient at risk for sepsis. Air embolism can occur when the intravenous tubing is disconnected from the catheter hub. Thrombosis occurs as a result of irritation of the vein from the central venous catheter. Pneumothorax is a complication of central venous catheter insertion.

During an admission assessment, the patient reports that he takes vitamin E supplements twice a day. The nurse should explain that taking vitamin E supplements twice a day 1) ensures healthy vision. 2) can lead to toxicity. 3) strengthens the immune system. 4) helps maintain body tissues.

Answer: 2) can lead to toxicity. Rationale: Vitamins are critical in building and maintaining body tissues, supporting the immune system to fight infection, and ensuring healthy vision. However when fat-soluble vitamins, such as vitamins A, D, E, and K, are supplemented in large doses, toxicity may occur.

An adult patient who is receiving a continuous enteral feeding at 80 ml/hr has a residual volume of 120 ml 6 hours after the last check. How should the nurse proceed? 1) Continue administering the enteral feeding. 2) Hold the enteral feeding and notify the provider immediately. 3) Hold the feeding for 1 hour, and recheck. 4) Hold the feeding for 2 hours, then resume the feeding.

Answer: 3) Hold the feeding for 1 hour, and recheck Rationale: The nurse should check enteral feeding residual every 4 to 6 hours. If residual is 10% greater than the formula flow rate for 1 hour (or alternatively, a total of 150 mL), the nurse should hold the feeding for 1 hour and recheck. If residual is still not within normal limits, she should notify the provider.

Which food provides the only animal source of carbohydrate? 1) Beef 2) Eggs 3) Milk 4) Chicken

Answer: 3) Milk Rationale: The only animal source of carbohydrate is lactose, the sugar contained in milk. Beef, eggs, and chicken do not provide a source of carbohydrate.

The nurse is caring for a patient who has multiple fractures from a skiing accident. To best promote bone growth, the nurse should encourage the patient to eat foods high in calcium and vitamin D. Which food selection by the client indicates an understanding of foods that are high in calcium? 1) Orange juice from concentrate 2) Cottage cheese 3) Tofu 4) Brie cheese

Answer: 3) Tofu Rationale: Firm tofu (½ cup serving) contains approximately 227 mg of calcium. This is an excellent dietary source of calcium. A ½ cup serving of orange juice from concentrate contains 27 mg of calcium. Brie cheese (1 oz) contains only 50 mg of calcium. A ½ cup serving of cottage cheese contains only 65 mg of calcium.

Which organ relies almost exclusively on glucose for energy? 1) Liver 2) Heart 3) Pancreas 4) Brain

Answer: 4) Brain Rationale: The brain relies almost exclusively on glucose for energy. The heart and liver do not. The pancreas produces insulin for glucose utilization but does not use glucose.

A patient has anemia. An appropriate goal for that the patient would be for him to increase his intake of which nutrient? 1) Calcium 2) Magnesium 3) Potassium 4) Iron

Answer: 4) Iron Rationale: Iron deficiency causes anemia; therefore, the nurse should encourage the patient with anemia to increase his intake of iron. Increasing calcium intake helps prevent osteoporosis. Magnesium supplementation may decrease the risk of hypertension and coronary artery disease in women. Potassium is essential for muscle contraction, acid-base balance, and blood pressure control.

The nurse is preparing an enteral feeding for a patient who will be receiving intermittent feedings via nasogastric tube for the first time. The patient is conscious. Which of the following is the priority intervention before administering this feeding? 1) Observe whether the patient can speak. 2) Inject air into the feeding tube while auscultating the stomach. 3) Aspirate stomach contents and measure residual volume. 4) Obtain an x-ray of the chest and abdomen.

Answer: 4) Obtain an x-ray of the chest and abdomen. Rationale: It is essential to verify that the NG tube is in the stomach and not the airway. The only reliable method among these four is to obtain an x-ray of the chest and abdomen. For subsequent feedings, however, it is not practical to obtain an x-ray each time, and presence of formula in the stomach complicates the pH readings. Therefore, the nurse should use a combination of bedside techniques, including observing appearance and measuring the amount of gastric residual volume, asking the patient to speak, and injecting air ("whoosh" test) while auscultating the stomach.

A nurse is instructing a group of overweight clients on the complications of obesity that develop when weight is not controlled through diet and exercise. Which lifestyle changes does the nurse emphasize? Select all that apply. A. "Begin a weight-training program for building muscle mass." B. "Consume a diet that is moderate in salt and sugar and low in fats and cholesterol." C. "Eat a variety of foods, especially grain products, vegetables, and fruits." D. "Engage in moderate physical activity for at least 30 minutes each day." E. "Foods eaten away from home tend to be higher in fat, cholesterol, and salt and lower in calcium than foods prepared at home." F. "Liquid dietary supplements can be substituted safely for solid food while attempting to lose weight."

Answers B, C, D, E Correct Feedback: Correct: Consuming a diet that is moderate in salt and sugar and low in fats and cholesterol is a smart strategy for a person who wants to lose weight. Correct: Eating a variety of foods, especially grain products, vegetables, and fruits, helps people achieve weight loss. These are foods that "burn" more calories as they are metabolized. Correct: Moderate physical activity for at least 30 minutes each day is a good idea for people who are trying to be healthy and/or to reduce their weight. Correct: True. Many foods eaten away from home tend to be higher in fat, cholesterol, and salt and lower in calcium than foods prepared at home. When dining "out," people can make smart choices, but they have to be educated and careful.

The client dx with anemia begins to complain of dyspnea when ambulating in the hall. Which intervention should the nurse implement first? A.Apply oxygen via nasal cannula B.Get a wheelchair for the client C.Assess the clients lung fields D.Assist the client when ambulating in the hall

B

The nurse and AP are caring for clients on a medical unit. Which task should the nurse delegate to the AP? A.Check on the bowel movements of a client dx with melena B.Take vital signs if a client who received blood the day before C.Evaluate the dietary intake of a client who has been noncompliant with eating D.Shave the client dx with severs hemolytic anemia

B

The client who is morbidly obese presents to the clinic before beginning a weight loss program. Which interventions should the nurse teach? (Select all that apply) A. Walk for 30 minutes three times a day. B. Determine situations that initiate eating behavior. C. Weigh at the same time every day. D. Limit sodium in the diet. E. Refer to a weight support group.

B,E The client should be aware of situations triggering the consumption of food when the client is not hungry, such as anger, boredom, and stress. Food-seeking behaviors are not associated only with hunger in the client who is obese. Weight loss support groups such as Weight Watchers or Take Off Pounds Sensibly are helpful to keep the client participating in a weight loss program.

A patient has a body mass index (BMI) of 27. How should the nurse explain this finding to the patient? A. normal weight B. overweight C. obese D. metabolic syndrome

B. A body mass index of 25-25.9 is considered overweight.

While reviewing the medical history, the nurse determines a patient is at risk for obesity. Which is the greatest risk factor for the development of obesity? A. Was adopted at two months of age. B. Does not engage in regular activity. C. Is allergic to chocolate and strawberries. D. Usual diet includes fat-food lunches twice a week.

B. Environmental influences and heredity contribute to the development of obesity. However, physical inactivity is the most important factor contributing to obesity.

The 22 year old female who is obese is discussing weight loss programs with the nurse. Which information should the nurse teach? A. Jog for two to three hours every day. B. Lifestyle behaviors must be modified. C. Eat one large meal every day in the evening. D. Eating 1,000 calories a day and don't take vitamins.

B. If lifestyle behaviors such as patterns of eating and daily exercise are not modified, the client who loses weight will regain the weight and usually more.

The ChooseMyPlate program includes guidelines for: A. Children younger than 2 years B. Balancing calories C. Increasing portion size D. Decreasing water consumption

B. Balancing calories The ChooseMyPlate program includes guidelines for balancing calories; decreasing portion size; increasing healthy foods; increasing water consumption; and decreasing fats, sodium, and sugars. These guidelines have been put forth for Americans over the age of 2 years.

The patient is to receive multiple medications via the nasogastric tube. The nurse is concerned that the tube may become clogged. To prevent this, the nurse: A. Irrigates the tube with 60 mL of water after all medications are given B. Checks with the pharmacy to find out if liquid forms of the medications are available C. Instills nonliquid medications without diluting D. Mixes all medications together to decrease the number of administrations

B. Checks with the pharmacy to find out if liquid forms of the medications are available Avoid crushed medication if liquid is available. Irrigate with 30 mL of water before and after each medication per tube. Dilute crushed medications if not liquid. Read pharmacological information on compatibility of drugs and formula before mixing medications.

The nurse is assessing a patient for nutritional status. In doing so, the nurse must: A. Choose a single objective tool that fits the patient's condition B. Combine multiple objective measures with subjective measures C. Forego the assessment in the presence of chronic disease D. Use the Mini Nutritional Assessment for pediatric patients

B. Combine multiple objective measures with subjective measures Using a single objective measure is ineffective in predicting risk of nutritional problems. Combine multiple objective measures with subjective measures related to nutrition to adequately screen for nutritional problems. Chronic disease and increased metabolic requirements are risk factors for the development of nutritional problems; these patients may be in critical need of this assessment. The Mini Nutritional Assessment was developed to use for screening older adults in home care programs, nursing homes, and hospitals.

When teaching a patient about current dietary guidelines for the general population, the nurse explains referenced daily intakes (RDIs) and daily reference values (DRVs), otherwise known as daily values. In providing this information, the nurse understands that daily values: A. Have replaced recommended daily allowances (RDAs) B. Have provided a more understandable format of RDAs for the public C. Are based on percentages of a diet consisting of 1200 kcal/day D.Are not usually easy to find computer experience is required

B. Have provided a more understandable format of RDAs for the public Daily values did not replace RDAs but provided a separate, more understandable format for the public. Daily values are based on percentages of a diet consisting of 2000 kcal/day; these values constitute the daily values used on food labels, which are easy for anyone to find. Computer experience is not required.

Before giving the patient an intermittent tube feeding, the nurse should: A. Make sure that the tube is secured to the gown with a safety pin B. Have the tube feeding at room temperature C. Inject air into the stomach via the tube and auscultate D. Place the patient in a supine position

B. Have the tube feeding at room temperature Cold formula causes gastric cramping and discomfort because the mouth and the esophagus do not warm the liquid. Do not use safety pins. Safety pins can become unfastened and may cause harm to the patient. Auscultation is no longer considered a reliable method for verification of tube placement because a tube inadvertently placed in the lungs, pharynx, or esophagus transmits sound similar to that of air entering the stomach. Place the patient in high-Fowler's position, or elevate the head of the bed at least 30 degrees to help prevent aspiration.

The patient with cardiovascular disease must be taught how to reduce the risk of cardiovascular disease by balancing calorie intake with exercise to maintain a healthy body weight. In addition to this, the nurse instructs the patient to: A. Eat fish at least 5 times per week B. Limit saturated fat to less than 7% C. Limit cholesterol to less than 200 mg/day D. Avoid high-fiber foods

B. Limit saturated fat to less than 7% AHA guidelines recommend limiting saturated fat to less than 7%, trans fat to less than 1%, and cholesterol to less than 300 mg/day. Diet therapy includes eating fish at least 2 times per week and eating whole grain high-fiber foods.

Knowing that protein is required for tissue growth, maintenance, and repair, the nurse must understand that for optimal tissue healing to occur, the patient must be in: A. Negative nitrogen balance B. Positive nitrogen balance C. Total dependence on protein for kcal production D. Neutral nitrogen balance

B. Positive nitrogen balance When intake of nitrogen is greater than output, the body is in positive nitrogen balance. Positive nitrogen balance is required for growth, normal pregnancy, maintenance of lean muscle mass and vital organs, and wound healing. Negative nitrogen balance occurs when the body loses more nitrogen than the body gains. Neutral nitrogen balance occurs when gain equals loss and is not optimal for tissue healing. Protein provides energy, but because of the essential role of protein in growth, maintenance, and repair, a diet needs to provide adequate kilocalories from nonprotein sources.

In providing diabetic teaching for a patient with type 1 diabetes mellitus, the nurse instructs the patient that: A. Insulin is the only consideration that must be taken into account B. Saturated fat should be limited to less than 7% of total calories C. Cholesterol intake should be greater than 200 mg/day D. Nonnutritive sweeteners can be used without restriction

B. Saturated fat should be limited to less than 7% of total calories The diabetic patient should limit saturated fat to less than 7% of total calories and cholesterol intake to less than 200 mg/day. Type 1 diabetes requires both insulin and dietary restrictions for optimal control. Nonnutritive sweeteners can be eaten as long as the recommended daily intake levels are followed.

In providing diet education for a patient on a low-fat diet, it is important for the nurse to understand that with few exceptions: A. Saturated fats are found mostly in vegetable sources B. Saturated fats are found mostly in animal sources C. Unsaturated fats are found mostly in animal sources D. Linoleic acid is a saturated fatty acid

B. Saturated fats are found mostly in animal sources Most animal fats have high proportions of saturated fatty acids, whereas vegetable fats have higher amounts of unsaturated and polyunsaturated fatty acids. Linoleic acid, an unsaturated fatty acid, is the only essential fatty acid in humans.

Dysphagia refers to difficulty when swallowing. Of the following causes of dysphagia, which is considered neurogenic? A. Myasthenia Gravis B. Stroke C. Candidiasis D. Muscular dystrophy

B. Stroke Stroke is the only cause of dysphagia in this list that is considered neurogenic. Myasthenia Gravis and muscular dystrophy are considered myogenic in origin, whereas candidiasis is considered obstructive.

The client was dx with iron-deficiency anemia is prescribed ferrous gluconate orally. Which should the nurse teach the client? A.Take Imodium, and anti diarrheal, OTC for diarrhea B.Limit exercise for several weeks until a tolerance is achieved C.The stools may be very dark, and this can mask blood D.Eat only red meats and organ meats for protein

C

The nurse is discharging a client dx with anemia. Which discharge instruction should the nurse teach? A.Take prescribed iron until it is completely gone B.Monitor P and BP at local pharmacy weekly C.Have complete blood count checked at the HCP's office D.Perform isometric exercise three times a week

C

The patient has been diagnosed with cardiovascular disease and placed on a low-fat diet. The patient asks the nurse, "How much fat should I have? I guess the less fat, the better." The nurse needs to explain that: A. Fats have no significance in health and the incidence of disease B. All fats come from external sources so can be easily controlled C. Deficiencies occur when fat intake falls below 10% of daily nutrition D. Vegetable fats are the major source of saturated fats and should be avoided

C. Deficiencies occur when fat intake falls below 10% of daily nutrition Deficiency occurs when fat intake falls below 10% of daily nutrition. Various types of fatty acids have significance for health and for the incidence of disease and are referred to in dietary guidelines. Linoleic acid and arachidonic acid are important for metabolic processes but are manufactured by the body when linoleic acid is available. Most animal fats have high proportions of saturated fatty acids, whereas vegetable fats have higher amounts of unsaturated and polyunsaturated fatty acids.

The patient is an 80-year-old male who is visiting the clinic today for his routine physical examination. The patient's skin turgor is fair, but he has been complaining of fatigue and weakness. The skin is warm and dry, pulse rate is 126 beats per minute, and urinary sodium level is slightly elevated. After assessment, the nurse should recommend that the patient: A. Decrease his intake of milk and dairy products to decrease the risk of osteoporosis B. Drink more grapefruit juice to enhance vitamin C intake and medication absorption C. Drink more water to prevent further dehydration D. Eat more meat because meat is the only source of usable protein

C. Drink more water to prevent further dehydration Thirst sensation diminishes, leading to inadequate fluid intake or dehydration. Symptoms of dehydration in older adults include confusion, weakness, hot dry skin, furrowed tongue, and high urinary sodium. Milk continues to be an important food for older woman and men, who need adequate calcium to protect against osteoporosis. After age 70, osteoporosis equally affects men and women. Caution older adults to avoid grapefruit and grapefruit juice because these will decrease absorption of many drugs. Some older adults avoid meats because of cost, or because they are difficult to chew. Cream soups and meat-based vegetable soups are nutrient-dense sources of protein.

To provide successful nutritional therapies to patients, the nurse must understand that: A. Patients will have to change diet preferences drastically to be successful B. The patient will tell the nurse when to change the plan of care C. Expectations of nurses frequently differ from those of the patient D. Nurses should never alter the plan of care regardless of outcome

C. Expectations of nurses frequently differ from those of the patient Expectations and health care values held by nurses frequently differ from those held by patients. Successful interventions and outcomes depend on recognition of this concept, in addition to nursing knowledge and skill. If ongoing nutritional therapies are not resulting in successful outcomes, patients expect nurses to recognize this fact and alter the plan of care accordingly. Working closely with patients enables the nurse to redefine expectations that are realistically met within the limits of conditions and treatments and to identify their dietary preferences and cultural beliefs.

In providing prenatal care to a patient, the nurse teaches the expectant mother that: A. Protein intake needs to decrease to preserve kidney function B. Calcium intake is especially important in the first trimester C. Folic acid is needed to help prevent birth defects and anemia D. The mother should take in as many extra vitamins and minerals as possible

C. Folic acid is needed to help prevent birth defects and anemia Folic acid intake is particularly important for DNA synthesis and growth of red blood cells. Inadequate intake may lead to fetal neural tube defects, anencephaly, or maternal megaloblastic anemia. Protein intake throughout pregnancy needs to increase to 60 g daily. Calcium intake is especially critical in the third trimester, when fetal bones are mineralized. Prenatal care usually includes vitamin and mineral supplementation to ensure daily intakes; however, pregnant women should not take additional supplements beyond prescribed amounts.

The nurse is providing nutrition teaching to a Korean patient. In doing so, the nurse must understand that the focus of the teaching should be on: A. Changing the patient's diet to a more conventional American diet B. Discouraging the patient's ethnic food choices C. Food preferences of the patient, including racial and ethnic choices D. Comparing the patient's ethnic preferences with American dietary choices

C. Food preferences of the patient, including racial and ethnic choices The nurse needs to make sure to consider the food preferences of different racial and ethnic groups, vegetarians, and others when planning diets. Initiation of a balanced diet is more important than conversion to what may be considered an American diet. Ethnic food choices may be just as nutritious as "American" choices. Foods should be chosen for their nutritive value and should not be compared with the "American" diet.

The patient has just been started on an enteral feeding and has developed diarrhea after being on the feeding for 2 hours. The most likely cause of the diarrhea would be: A. Clostridium difficile B. Antibiotic therapy C. Formula intolerance D. Bacterial contamination

C. Formula intolerance Hyperosmolar formulas can cause diarrhea. If that is the case, the solution is to lower the rate, dilute the formula, or change to an isotonic formula. Antibiotics destroy normal intestinal flora and disturb the internal ecology, allowing for Clostridium difficile toxin buildup. However, this takes time, and no indication suggests that this patient was on antibiotics. Proximity to the start of the enteral feedings is more suspicious. Bacterial contamination of the feeding usually occurs when feedings are left hanging for longer than 8 hours.

The patient is on PN and is lethargic. He has been complaining of thirst and headache and has had increased urination. Which of the following problems would cause these symptoms? A. Electrolyte imbalance B. Hypoglycemia C. Hyperglycemia D. Hypercapnia

C. Hyperglycemia Signs and symptoms of hyperglycemia are thirst, headache, lethargy, and increased urination. Electrolyte imbalance is marked by changes in Na, Ca, K, Cl, PO4, Mg, and CO2 levels. These have to be monitored closely when patients are on PN. Hypercapnia increases oxygen consumption and increases CO2 levels. Ventilator-dependent patients are at greatest risk for this. Hypoglycemia is characterized by diaphoresis, shakiness, confusion, and loss of consciousness.

To counter obesity in adolescents, increasing physical activity is often more important than curbing intake. Sports and regular, moderate to intense exercise necessitate dietary modifications to meet increased energy needs for adolescents. The nurse understands that these modifications include: A. Decreasing carbohydrates to 25% to 30% of total intake B. Decreasing protein intake to .75 g/kg/day C. Ingesting water before and after exercise D. Providing vitamin and mineral supplements

C. Ingesting water before and after exercise Adequate hydration is very important for all athletes. They need to ingest water before and after exercise to prevent dehydration, especially in hot, humid environments. Carbohydrates, both simple and complex, are the main source of energy, providing 55% to 60% of total daily kilocalories. Protein needs increase to 1.0 to 1.5 g/kg/day. Vitamin and mineral supplements are not required, but intake of iron-rich foods is required to prevent anemia.

In creating a plan of care to meet the nutritional needs of the patient, the nurse needs to explore the patient's feelings about weight and food. The nurse must do this to: A. Determine which category of plan to use B. Set realistic goals for the patient C. Mutually plan goals with patient and team D. Prevent the need for a dietitian consult

C. Mutually plan goals with patient and team Mutually planned goals negotiated by patient, registered dietitian, and nurse ensure success. Individualized planning cannot be overemphasized. Preplanned and categorical care plans are not effective unless they are individualized to meet patient needs. It is important to explore patients' feelings about weight and food to help them set realistic and achievable goals. The nurse does not set goals for the patient. The plan should reflect the combined effort of patient, nurse, and dietitian, so a dietitian consult is required.

The patient is having at least 75% of his nutritional needs met by enteral feeding, so the physician has ordered the PN to be discontinued. However, the nurse notices that the PN infusion has fallen behind. The nurse should: A. Increase the rate to get the volume caught up before discontinuing B. Stop the infusion and hang a normal saline drip in place C. Taper the PN infusion gradually D. Hang 5% dextrose if the PN runs out

C. Taper the PN infusion gradually Sudden discontinuation of PN can cause hypoglycemia. PN must be tapered off. Usually, 10% dextrose is infused when PN solution is suddenly discontinued. The same is true if the PN runs out. Too rapid administration of hypertonic dextrose (PN) can result in an osmotic diuresis and dehydration. If an infusion falls behind schedule, the nurse should not increase the rate in an attempt to catch up.

The nurse is caring for a patient who will be receiving PN. To reduce the risk of developing sepsis, the nurse: A. Takes down a running bag of TPN after 36 hours B. Runs lipids for no longer than 24 hours C. Wears a sterile mask when changing the CVC dressing D. Wears clean gloves when changing the CVC dressing

C. Wears a sterile mask when changing the CVC dressing During CVC dressing changes, always use a sterile mask and gloves, and assess insertion sites for signs and symptoms of infection. To avoid infection, change the TPN infusion tubing every 24 hours, and do not hang a single container of PN for longer than 24 hours or lipids longer than 12 hours.

In general, when energy requirements are completely met by kilocalorie (kcal) intake in food: A. Weight increases B. Weight decreases C. Weight does not change D. Kilocalories are not a factor

C. Weight does not change In general, when energy requirements are completely met by kilocalorie (kcal) intake in food, weight does not change. When kilocalories ingested exceed a person's energy demands, the individual gains weight. If kilocalories ingested fail to meet a person's energy requirement, the individual loses weight. Kilocalories are a factor.

At present, the most reliable method for verification of placement of small-bore feeding tubes is: A. Auscultation B. Aspiration of contents C. X-ray D. pH testing

C. X-ray At present, the most reliable method for verification of placement of small-bore feeding tubes is x-ray examination. Aspiration of contents and pH testing are not infallible. The nurse would need a more precise indicator to help differentiate the source of tube feeding aspirate. Auscultation is no longer considered a reliable method for verification of tube placement because a tube inadvertently placed in the lungs, pharynx, or esophagus transmits sound similar to that of air entering the stomach.

Priority Decision: Before selecting a weight reduction plan with an obese patient, what is most important for the nurse to first assess? a. The patient's motivation to lose weight b. The length of time that the patient has been obese c. Whether financial considerations will affect the patient's choices d. The patient's anthropometric measures of height, weight, BMI, waist-to-hip ratio, and skinfold thickness

Correct answer: a Rationale: Motivation is essential. Focus on the reasons for wanting to lose weight. The rest of the options will asset in planning the weight loss if the patient is motivated.

When teaching a patient about weight reduction diets, the nurse teaches the patient that an appropriate single serving of a food is a. a 6-inch bagel. b. 1 cup of chopped vegetables. c. a piece of cheese the size of three dice. d. a chicken breast the size of a deck of cards.

Correct answer: d Rationale: A chicken breast the size of a deck of cards is about 3 oz, a recommended portion size of meat. Other normal portions include a 3-inch bagel, 1/2 cup of chopped vegetables, and a piece of cheese the size of six dice.

The nurse is caring for a patient who is 5'6" tall and weighs 186 lb. The nurse has discussed reasonable weight loss goals and a low-calorie diet with the patient. Which statement made by the patient indicates a need for further teaching? a. "I will limit intake to 500 calories a day." b. "I will try to eat very slowly during mealtimes." c. "I'll try to pick foods from all of the basic food groups." d. "It's important for me to begin a regular exercise program."

Correct answer: a Rationale: Limiting intake to 500 calories per day is not indicated for this patient, and the severe calorie energy restriction would place this patient at risk for multiple nutrient deficiencies. Decreasing caloric intake at least 500 to 1000 calories a day is recommended for weight loss of one to two pounds per week. The other options show understanding of the teaching.

The nurse instructs an obese 22-year-old man with a sedentary job about the health benefits of an exercise program. The nurse evaluates that teaching is effective when the patient makes which statement? a. "The goal is to walk at least 10,000 steps every day of the week." b. "Weekend aerobics for 2 hours is better than exercising every day." c. "Aerobic exercise will increase my appetite and result in weight gain." d. "Exercise causes weight loss by decreasing my resting metabolic rate."

Correct answer: a Rationale: A realistic activity goal is to walk 10,000 steps a day. Increased activity does not promote an increase in appetite or lead to weight gain. Exercise should be done daily, preferably 30 minutes to an hour a day. Exercise increases metabolic rate.

A complete nutritional assessment including anthropometric measurements is important for the patient who a. has a BMI of 25.5 kg/m2. b. complains of frequent nocturia. c. reports a 5-year history of constipation. d. reports an unintentional weight loss of 10 lb in 2 months.

Correct answer: d Rationale: A loss of more than 5% of usual body weight over 6 months, whether intentional or unintentional, is a critical indicator for further assessment.

A 50-year-old African American woman has a BMI of 35 kg/m2, type 2 diabetes mellitus, hypercholesterolemia, and irritable bowel syndrome (IBS). She is seeking assistance in losing weight, because, "I have trouble stopping eating when I should, but I do not want to have bariatric surgery." Which drug therapy should the nurse question if it is prescribed for this patient? a. Orlistat (Xenical) b. Locaserin (Belviq) c. Phentermine (Adipex-P) d. Phentermine and topiramate (Qsymia)

Correct answer: a Rationale: Orlistat (Xenical), which blocks fat breakdown and absorption in the intestine, produces some unpleasant GI side effects. This drug would not be appropriate for someone with IBS. Locaserin (Belviq) suppresses the appetite and creates a sense of satiety that may be helpful for this patient. Phentermine (Adipex-P) needs to be used for a limited period of time (3 months or less). Qsymia is a combination of two drugs, phentermine and topiramate. Phentermine is a sympathomimetic agent that suppresses appetite and topiramate induces a sense of satiety.

A patient has been on a 1000-calorie diet with a daily exercise routine. In 2 months, the patient has lost 20 lb (9kg) toward a goal of 50 lb (23 kg) but is now discouraged that no weight has been lost in the last 2 weeks. What should the nurse tell the patient about this? a. Plateaus where no weight is lost normally occur during a weight-loss program. b. A weight considered by the body to most efficient for functioning has been reached. c. A return to former eating habits is the most common cause of not continuing to lose weight. d. A steady weight may be due to water gain from eating foods high in sodium.

Correct answer: a Rationale: Plateau periods during which no weight is lost are normal occurrences during weight reduction and may last for several days to several weeks but weight loss will resume if the prescribed weight reduction plan is continued. Weight loss may stop if former eating habits are resumed but this not the most common cause of plateaus.

A patient asks the nurse about taking phentermine and topiramate (Qsymia) for weight loss. To avoid side effects, it is important for the nurse to determine whether the patient has a history of a. glaucoma. b. hypertension. c. valvular heart disease. d. irritable bowel disease.

Correct answer: a Rationale: Qsymia is a combination of phentermine and topiramate. It must not be used in patients with glaucoma or hyperthyroidism.

The percentage of daily calories for a healthy individual consists of: a. 50% carbohydrates, 25% protein, 25% fat, and <10% of fat from saturated fatty acids. b. 65% carbohydrates, 25% protein, 25% fat, and >10% of fat from saturated fatty acids. c. 50% carbohydrates, 40% protein, 10% fat, and <10% of fat from saturated fatty acids. d. 40% carbohydrates, 30% protein, 30% fat, and >10% of fat from saturated fatty acids.

Correct answer: a Rationale: The 2005 Dietary Guidelines for Americans recommend that 45% to 65% of total calories should come from carbohydrates. Ideally, 10% to 35% of daily caloric needs should come from protein. Individuals should limit their fat intake to 20% to 35% of total calories. Additional recommendations focus on the type of fat consumed because diets high in excess calories, usually in the form of fats, contribute to the development of obesity. Individuals should consume less than 10% of calories from saturated fatty acids, limit intake of fat and oils high in trans fatty acids, and should limit intake of dietary cholesterol to 300 mg/day.

During the initial postoperative period following bariatric surgery, the nurse recognizes the importance of monitoring obese patients for respiratory insufficiency based on what knowledge? a. The body stores anesthetics in adipose tissue. b. Postoperative pain may cause a decreased respiratory rate. c. Intubation may be difficult because of extra chin skinfolds. d. The patient's head must remain flat for a minimum of 2 hours postprocedure.

Correct answer: a Rationale: The body stores anesthetics in adipose tissue, placing patients with excess adipose tissue at risk for re-sedation. As adipose cells release anesthetics back into the bloodstream, the patient may become sedated after surgery, increasing the risk of hypoventilation and resultant respiratory insufficiency. Difficult intubation does not cause respiratory insufficiency. Pain usually increases respiratory rate. The patient's head should be elevated after bariatric surgery to decrease abdominal pressure and facilitate respirations.

The nurse confirms initial placement of a blindly inserted smallbore NG feeding tube by a. x-ray. b. air insufflation. c. observing patient for coughing. d. pH measurement of gastric aspirate.

Correct answer: a Rationale: The nurse should obtain x-ray confirmation to determine whether a blindly placed nasogastric or orogastric tube (small bore or large bore) is properly positioned in the gastrointestinal tract before administering feedings or medications.

The nurse teaches a 50-year-old woman who has a body mass index (BMI) of 39 kg/m2 about weight loss. Which dietary change would be appropriate for the nurse to recommend to this patient? a. Decrease fat intake and control portion size b. Increase vegetables and decrease fluid intake c. Increase protein intake and avoid carbohydrates d. Decrease complex carbohydrates and limit fiber

Correct answer: a Rationale: The safest dietary guideline for weight loss is to decrease caloric intake by maintaining a balance of nutrients and adequate hydration while controlling portion size and decreasing fat intake.

The nurse is caring for a 45-year-old woman with a herniated lumbar disc. The patient realizes that weight loss is necessary to lessen back strain. The patient is 5'6" tall and weighs 186 lb (84.5 kg) with a body mass index (BMI) of 28 kg/m2. The nurse explains to the patient that this measurement places her in which of the following weight categories? a. Normal weight b. Overweight c. Obese d. Severely obese

Correct answer: b Rationale: A normal BMI is 18.5 to 24.9 kg/m2, whereas a BMI of 25 to 29.9 kg/m2 is considered overweight. A BMI of 30.0-39.9 is considered obese, and a BMI of 40 or greater is severely obese.

A woman is 5 ft, 6 in (166 cm) tall and weighs 200 lb (90.9 kg) with a waist-to-hip ratio of 0.7. The nurse counsels the patient with the knowledge that the patient is at greatest risk for a. heart disease. b. osteoporosis. c. diabetes mellitus. d. endometrial cancer.

Correct answer: b Rationale: A patient who is obese (BMI of 32.2) but has a waist-to-hip ratio of less than 0.8, indicating gynoid obesity, has an increased risk for osteoporosis. The other conditions are risks associated with android obesity.

A patient is receiving peripheral parenteral nutrition. The parenteral nutrition solution is completed before the new solution arrives in the unit. The nurse administers a. 20% intralipids. b. 5% dextrose solution. c. 0.45% normal saline solution. d. 5% lactated Ringer's solution.

Correct answer: b Rationale: If a peripheral parenteral nutrition (PPN) formula bag empties before the next solution is available, a 5% dextrose solution (based on the amount of dextrose in the peripheral PN solution) may be administered to prevent hypoglycemia

During starvation, the order in which the body obtains substrate for energy is a. visceral protein, skeletal protein, fat, glycogen. b. glycogen, skeletal protein, fat stores, visceral protein. c. visceral protein, fat stores, glycogen, skeletal protein. d. fat stores, skeletal protein, visceral protein, glycogen.

Correct answer: b Rationale: Initially, the body selectively uses carbohydrates (e.g., glycogen) rather than fat and protein to meet metabolic needs. These carbohydrate stores, found in the liver and muscles, are minimal and may be totally depleted within 18 hours. After carbohydrate stores are depleted, skeletal protein begins to be converted to glucose for energy. Within 5 to 9 days, body fat is fully mobilized to supply much of the needed energy. In prolonged starvation, up to 97% of calories are provided by fat, and protein is conserved. Depletion of fat stores depends on the amount available, but fat stores typically are used up in 4 to 6 weeks. After fat stores are used, body or visceral proteins, including those in internal organs and plasma, can no longer be spared and rapidly decrease because they are the only remaining body source of energy available.

Priority Decision: When medications are used in the treatment of obesity, what is most important for the nurse to teach the patient? a. Over-the-counter (OTC) diet aids are safer than other agents and con be useful in controlling appetite. b. Drugs should be used only as adjuncts to a diet and exercise program as treatment for a chronic condition. c. All drugs used for weight control are capable of altering central nervous system (CNS) function and should be used with caution. d. The primary effect of the medications is psychologic, controlling the urge to eat in response to stress or feelings of rejection.

Correct answer: b Rationale: Medications are used only as adjuncts to diet and exercise programs in the treatment of obesity. Drugs do not cure obesity; without changes in food intake and physical activity, weight gain will occur when the medications are discontinued. The medications used work in a variety of ways to control appetite but over-the-counter drugs are probably the least effective and most abused of these drugs.

Which patient is at highest risk for complications of obesity? a. A 30-year-old woman who is 5 ft (151 cm) tall, weighs 140 lb (63 kg), and carries weight in her thighs. b. A 56-year-old woman with a BMI of 38 kg/m2, a waist measurement of 38 in (96 cm), and a hip measurement of 36 in (91 cm) c. A 42-year-old man with a waist measurement of 36 in (91 cm) and a hip measurement of 36 in (91 cm) who is 5 ft, 6 in (166 cm) tall and weighs 150 lb (68.2 kg) d. A 68-year-old man with a waist measurement of 38 in (96 cm) and a hip measurement of 42 in (76 cm) who is 5 ft, 11 in (179 cm) tall and weighs 200 lb (90.9 kg)

Correct answer: b Rationale: The 56-year-old woman has a body mass index (BMI) of 38 kg/m2 (obese, Class II) with a waist-to-hip ratio of 1.1 with android obesity and is more at risk (very high) than the other patients. The 30-year-old woman has the least risk with a BMI of 27.3 kg/m2 (overweight) and gynoid shape. The 42-year-old man has a BMI of 24.2 kg/m2 (normal weight) with one risk factor in the waist-to-hip ratio of 1.0 and the 68-year-old man has a BMI of 27.9 kg/m2 (overweight) with a waist-to-hip ratio of 0.9.

The nurse has completed initial instruction with a patient regarding a weight loss program. The nurse determines that the teaching has been effective when the patient makes which statement? a. "I plan to lose 4 lb a week until I have lost the 60-pound goal." b. "I will keep a diary of weekly weights to illustrate my weight loss." c. "I will restrict my carbohydrate intake to less than 30 g/day to maximize weight loss." d."I should not exercise more than my program requires since increased activity increases the appetite."

Correct answer: b Rationale: The patient should monitor and record weight once per week. This prevents frustration at the normal variations in daily weights and may help the patient to maintain motivation to stay on the prescribed diet. Weight loss should occur at a rate of 1 to 2 lb/week. The diet should be well balanced rather than lacking in specific components that may cause an initial weight loss but is not usually sustainable. Exercise is a necessary component of any successful weight loss program.

Which female patient is most likely to have metabolic syndrome? a. BP 128/78 mm Hg, triglycerides 160 mg/dL, fasting blood glucose 102 mg/dL b. BP 142/90 mm Hg, high-density lipoproteins 45 mg/dL, fasting blood glucose 130 mg/dL c. Waist circumference 36 in, triglycerides 162 mg/dL, high-density lipoproteins 55 mg/dL d. Waist circumference 32 in, high-density lipoproteins 38 mg/dL, fasting blood glucose 122 mg/dL

Correct answer: b Rationale: Three of the following five measures are needed for a woman to be diagnosed with metabolic syndrome: waist circumference >35 in, triglycerides >150 mg/dL, high-density lipoproteins <50 mg/dL, BP >130 mm Hg systolic or >85 mm Hg diastolic, fasting blood glucose >110 mg/dL. Although the other options have some abnormal measures, none has all three measures in the diagnostic ranges. The criteria for metabolic syndrome for both women and men are listed in Table 41-10.

Which statement about obesity is explained by genetics? a. Older obese patients have exacerbated changes of aging. b. Android body shape and weight gain are influenced by genetics. c. White Americans have a higher incidence of obesity than African Americans. d. Men have a harder time losing weight, as they have more muscle mass than women.

Correct answer: b Rationale: Twin studies and studies with adopted children have shown that body shape and weight gain are influenced by genetics but more research is needed. Older obese people do have exacerbated aging problems related to declines in physical function. African Americans and Hispanics have a higher incidence of obesity than whites. Women have a higher incidence of obesity and more difficulty losing weight than men because women have a higher percentage of metabolically less-active fat.

In developing a weight reduction program with a 45-year-old female patient who weighs 197 lb, the nurse encourages the patient to set a weight loss goal of how many pounds in 4 weeks? a. 1-2 b. 3-5 c. 4-8 d. 5-10

Correct answer: c Rationale: A realistic weight loss goal for patients is 1 to 2 lb/wk, which prevents the patient from becoming frustrated at not meeting weight loss goals.

In developing an effective weight reduction plan for an overweight patient who states a willingness to try to lose weight, it is most important for the nurse to first assess which factor? a. The length of time the patient has been obese b. The patient's current level of physical activity c. The patient's social, emotional, and behavioral influences on obesity d. Anthropometric measurements, such as body mass index and skinfold thickness

Correct answer: c Rationale: Eating patterns are established early in life, and eating has many meanings for people. To establish a weight reduction plan that will be successful for the patient, the nurse should first explore the social, emotional, and behavioral influences on the patient's eating patterns. The duration of obesity, current physical activity level, and current anthropometric measurements are not as important for the weight reduction plan.

Priority Decision: During care of the severely obese patient, what is most important for the nurse to do? a. Avoid reference to the patient's weight to avoid embarrassing the patient. b. Emphasize to the patient how important it is to lose weight to maintain health. c. Plan for necessary modifications in equipment and nursing techniques before initiating care. d. Recognize that a full assessment of each body system might not be possible because of numerous layers of skinfolds.

Correct answer: c Rationale: Special considerations are needed for the care of the severely obese patient because most hospital units are not prepared with beds, chairs, BP cuffs, and other equipment that will need to be used with the very obese patient. Consideration of all aspects of care should be made before implementing care for the patient, including extra time and perhaps assistance for positioning, physical assessment, and transferring the patient.

Which statement best describes the etiology of obesity? a. Obesity primarily results from a genetic predisposition. b. Psychosocial factors can override the effects of genetics in the etiology of obesity. c. Obesity is the result of complex interactions between genetic and environmental factors. d. Genetic factors are more important than environmental factors in the etiology of obesity.

Correct answer: c Rationale: The cause of obesity involves significant genetic and biologic susceptibility factors that are highly influenced by environmental and psychosocial factors.

Which explanation about weight reduction should be included when teaching the obese patient and her obese husband? a. Weight gain is caused by psychologic factors. b. Daily weighing is recommended to monitor weight loss. c. Fat is not burned until the glycogen-water pool is depleted. d. Men lose weight less quickly than women because they have a higher percentage of metabolically less-active fat.

Correct answer: c Rationale: With reducing diets that severely restrict carbohydrates, the body's glycogen stores become depleted within a few days. The glycogen normally binds to water in fat cells and it is this water loss that causes weight loss in the first few days. Fat is not burned until the glycogen-water pool is depleted. Although psychosocial components (i.e., using food for comfort or reward and inability to buy high-nutritional quality food) may have an influence on weight gain, these factors along with lack of physical exercise, underestimation of portion size, and genetics contribute to weight gain. Weekly weighing is recommended as a more reliable indicator of weight loss because daily weighing shows frequent fluctuation from retained water (including urine) and elimination of feces. Men are able to lose weight more quickly than women because women have a higher percentage of metabolically less-active fat.

In the immediate postoperative period a nurse cares for a severely obese 72-year-old man who had surgery for repair of a lower leg fracture. Which assessment would be most important for the nurse to make? a. Cardiac rhythm b. Surgical dressing c. Postoperative pain d. Oxygen saturation

Correct answer: d Rationale: After surgery an older and/or severely obese patient should be closely monitored for oxygen desaturation. The body stores anesthetics in adipose tissue, placing patients with excess adipose tissue (e.g., obesity, older) at risk for resedation. As adipose cells release anesthetic back into the bloodstream, the patient may become sedated after surgery. This may depress the respiratory rate and result in a drop in oxygen saturation.

At the first visit to the clinic, the female patient with a BMI of 29 kg/m2 tells the nurse that she does not want to become obese. Which question used for assessing weight issues is the most important question for the nurse to ask? a. "What factors contributed to your current body weight?" b. "How is your overall health affected by your body weight?" c. "What is your history of gaining weight and losing weight?" d. "In what ways are you interested in managing your weight differently?"

Correct answer: d Rationale: Asking the patient about her desire to manage her weight in a different manner helps the nurse determine the patient's readiness for learning, degree of motivation, and willingness to change lifestyle habits. The nurse can help the patient set realistic goals. This question will also lead to discussing the patient's history of gaining and losing weight and factors that have contributed to the patient's current weight. The patient may be unaware of the overall health effects of her body weight, so this question is not helpful at this time.

The obesity classification that is most often associated with cardiovascular health problems is a. primary obesity. b. secondary obesity. c. gynoid fat distribution. d. android fat distribution.

Correct answer: d Rationale: Individuals with fat located primarily in the abdominal area (i.e., whose body is apple-shaped) are at greater risk for obesity-related complications (e.g., heart disease) than are those whose fat is primarily located in the upper legs (i.e., whose body is pear-shaped). Individuals whose fat is distributed over the abdomen and upper body (i.e., neck, arms, and shoulders) are classified as having android obesity.

The nurse has completed initial instruction with a patient regarding a weight-loss program. Which patient comment indicates to the nurse that the teaching has been effective? a. "I will keep a diary of daily weight to illustrate my weight loss." b. "I plan to lose 4 lb a week until I have lost the 60 lb I want to lose." c. "I should not exercise more than what is required so I don't increase my appetite." d. "I plan to join a behavior modification group to help establish long-term behavior changes."

Correct answer: d Rationale: People who have undergone behavior therapy are more successful in maintaining weight losses over time because most programs deemphasize the diet, focus on how and when the person eats and education, and provide support from others. Weighing daily is not recommended and plateaus may not allow for consistent weight loss. A goal for weight loss must be set and 1 to 2 pounds a week is realistic. A more rapid loss often causes skin and underlying tissue to lose elasticity and become flabby folds of tissue. Exercising more often depresses appetite and exercise need not be limited.

A patient with anorexia nervosa shows signs of malnutrition. During initial refeeding, the nurse carefully assesses the patient for a. hyperkalemia. b. hypoglycemia. c. hypercalcemia. d. hypophosphatemia.

Correct answer: d Rationale: Refeeding syndrome is characterized by fluid retention, electrolyte imbalances (e.g., hypophosphatemia, hypokalemia, hypomagnesemia), and hyperglycemia. Conditions that predispose patients to refeeding syndrome include long-standing malnutrition states such as those induced by chronic alcoholism, vomiting and diarrhea, chemotherapy, and major surgery. Refeeding syndrome can occur any time a malnourished patient is started on aggressive nutritional support. Hypophosphatemia is the hallmark of refeeding syndrome, and it is associated with serious outcomes, including cardiac dysrhythmias, respiratory arrest, and neurologic disturbances (e.g., paresthesias).

The best nutritional therapy plan for a person who is obese is a. the Zone diet. b. the Atkins diet. c. Sugar Busters. d. foods from the basic food groups.

Correct answer: d Rationale: Restricted food intake is a cornerstone for any weight loss or maintenance program. A good weight loss plan should include foods from the basic food groups.

Which patient has the highest morbidity risk? a. Male 6 ft. 1 in. tall, BMI 29 kg/m2 b. Female 5 ft. 6 in. tall, weight 150 lb. c. Male with waist circumference 46 in. d. Female 5 ft. 10 in. tall, obesity Class III

Correct answer: d Rationale: The patient in Class III obesity has the highest risk for disease because Class III denotes severe obesity or a BMI greater than 40 kg/m2. The patient with waist circumference 46 in. has a high risk for disease, but without the BMI or obesity class, a more precise determination cannot be made. The female who is 5 ft. 6 in. tall has a normal weight for her height. The male patient who is over 6 ft. tall is overweight, which increases his risk of disease, but a more precise determination cannot be made without the waist circumference.

Wich of the following criteria must be met for a diagnosis of metabolic syndrome (select all that apply)? a. Hypertension b. Elevated triglycerides c. Elevated plasma glucose d. Increased waist circumference e. Decreased low-density lipoproteins

Correct answers: a, b, c, d Rationale: Three of the following five criteria must be met for a diagnosis of metabolic syndrome: • Waist circumference of 40 inches or more in men and 35 inches or more in women • Triglyceride levels higher than 150 mg/dL, or need for drug treatment for elevated triglyceride levels • High-density lipoprotein (HDL) cholesterol levels lower than 40 mg/dL in men and lower than 50 mg/dL in women, or need for drug treatment for reduced HDL cholesterol levels • Blood pressure: 130 mm Hg or higher systolic or 85 mm Hg or higher diastolic, or need for drug treatment for hypertension • Fasting blood glucose level of 110 mg/dL or higher, or need for drug treatment for elevated glucose levels

Normally, which hormones and peptides affect appetite (select all that apply)? a. Leptin b. Insulin c. Ghrelin d. Peptide YY e. Neuropeptide Y f. Cholecystokinin

Correct answers: a, b, c, d, e, f Rationale: Normally ghrelin and neuropeptide Y stimulate appetite. Leptin suppresses appetite and hunger. Insulin decreases appetite. Peptide YY and cholecystokinin inhibit appetite by slowing gastric emptying and sending satiety signals to the hypothalamus.

Priority Decision: The nurse is teaching a moderately obese woman interventions for the management of obesity. Initially, which strategies will support restricting dietary intake to below energy requirements (select all that apply)? a. Limit alcohol b. Rest when fatigued c. Determine portion sizes d. 1800- to 2200-calorie diet e. Attend Overeaters Anonymous

Correct answers: a, c Rationale: To restrict dietary intake so that it is below energy requirements, the moderately obese woman should limit or avoid alcohol intake because it increases caloric intake and has low nutritional value. Portion sizes have increased over the years and are larger than they should be. Teach the patient to determine portion sizes by weight or learn equivalencies such as that a serving of fruit is the size of a baseball. A progressive exercise program will increase energy requirements and a diet with an initial 800- to 1200-calorie limit would decrease calorie intake. Overeaters Anonymous would not restrict dietary intake below energy requirements, although it may offer support for the patient.

Health risks associated with obesity include (select all that apply) a. colorectal cancer. b. rheumatoid arthritis. c. polycystic ovary syndrome. d. nonalcoholic steatohepatitis. e. systemic lupus erythematosus.

Correct answers: a, c, d Rationale: Health risks associated with obesity include cardiovascular disease (related to increased low-density lipoprotein levels, increased triglyceride levels, and decreased high-density lipoprotein levels), hypertension, sleep apnea, obesity hypoventilation syndrome, reduced chest wall compliance, increased work of breathing, decreased total lung capacity and functional residual capacity, type 2 diabetes mellitus (i.e., hyperinsulinemia and insulin resistance), osteoarthritis, hyperuricemia, gout, gastroesophageal reflux disease, gallstones, nonalcoholic steatohepatitis, fatty liver and cirrhosis, cancer (mainly breast, endometrial, kidney, colorectal, pancreas, esophagus, and gallbladder), psychosocial problems (employment, education, and health care), low self-esteem, withdrawal from social interactions, and major depression.

Which teaching points are important when providing information to a patient with metabolic syndrome (select all that apply)? a. Stop smoking. b. Monitor weight daily. c. Increase level of activity. d. Decrease saturated fat intake. e. Reduce weight and maintain lower weight. f. Check blood glucose each morning prior to eating.

Correct answers: a, c, d, e Rationale: Patients with metabolic syndrome need to lower their risk factors by reducing and maintaining weight, increasing physical activity, establishing healthy diet habits, and smoking cessation. Some patients with metabolic syndrome are diabetic and would need to monitor glucose levels frequently. When monitoring weight reduction, it is recommended to check weight weekly, not daily.

In preparing to care for the obese patient with cancer, what physiologic problems is this patient at a greater risk for having (select all that apply)? a. Tinnitus b. Fractures c. Sleep apnea d. Trousseau's sign e. Type 2 diabetes mellitus f. Gastroesophageal reflux disease (GERD)

Correct answers: c, e, f Rationale: Obese patients are at a higher risk for cancer, sleep apnea and sleep deprivation, type 2 diabetes mellitus, gastroesophageal reflux disease (GERD), nonalcoholic steatohepatits, osteoarthritis, and cardiovascular problems. The other options are not related to obesity.

The nurse is identifying a diagnosis appropriate for a patient with obesity. Which diagnosis is the priority for a patient with a BMI of 30.4kg/m2 and a waist-to-hip ratio of 1.1? A. Ineffective Coping B. Deficient Knowledge: Diet C. Health-Seeking Behaviors: Weight Loss D. Risk for impaired Tissue perfusion: Cardiac

D. A BMI greater than 25 and central obesity as indicated by a waist-hip ratio of 1 or greater tend to have more intraabdominal fat and higher levels of circulating free fatty acids.

The female client is more than 10% over ideal body weight. Which nursing intervention should the nurse implement first? A. Ask the client why she is eating too much. B. Refer the client to a gymnasium for exercise. C. Have the client set a realistic weight loss goal. D. Determine the client's eating patterns.

D. Determining the client's eating patterns and what triggers the client to eat-stress or boredom, for example-and where and when the client consumes most of the calories-snacking in front of the TV at night, for example- is needed to assist the client to change eating behaviors.

The 36 year old female client diagnosed with anorexia nervosa tells the nurse "I am so fat. I won't be able to eat today" Which response by the nurse is most appropriate? A. "Can you tell me why you think you are fat?" B. "You are skinny. many women wish they had you problem." C. "If you don't eat we will have to restrain you and feed you." D. "Not eating might cause physical problems."

D. It is a factual statement to the client about the possible results if the client about the possible results if the client refuses nourishment.

A patient has a waist-to-hip ratio of 0.5. The nurse realizes that this patient is at risk for developing what disorder? A. gastrointestinal dysfunction B. hyperinsulinemia C. heart disease D. obesity

D. Lower body obesity may be more difficult to treat.

A patient on a reduced-calorie diet asks the nurse what she can do to lose weight faster, because most weeks she loses no more than 0.5lb. "At this rate, it will take me years to get to my goal!" What should the nurse respond to this patient? A. "Let's reevaluate your long-term goal. Perhaps it was set too low for you." B. "You sound frustrated. Would you like to take some time off from your diet and exercise plan?" C. "Perhaps we should look into a diet supplement since you are unable to stick with your prescribed diet plan" D. "A pound of body fat equals 3500 calories. Let's reevaluate your diet and exercise plan for calorie intake and expenditure."

D. The nurse should assist the patient to create attainable goals that incorporate achievement of improved health outcomes.

The nurse is concerned that a postoperative patient is at risk for malnutrition. Which intervention would be a priority to prevent malnutrition in this patient? A. Daily weights B. aggressive pain management C. Maintaining intravenous flow D. Requesting early restoration of oral intake

D. The stress of surgery produces a state of hypermetabolism and catabolism, which increases energy expenditure and nutrient needs, resulting in protein-calorie malnutrition. Beginning oral intake of nutrients as soon as possible after surgery is the best way to prevent the development of this type of malnutrition.

In determining kcal expenditure, the nurse knows that carbohydrates and proteins provide 4 kcal of energy per gram ingested. The nurse also knows that fats provide _____ kcal per gram. A. 3 B. 4 C. 6 D. 9

D. 9 Fats (lipids) are the most calorie-dense nutrient, providing 9 kcal per gram.

Patients who are unable to digest or absorb enteral nutrition benefit from parenteral nutrition (PN). However, the goal to move toward use of the GI tract is constant because PN: A. Can be given only in the hospital setting B. Cannot be used in patients in highly stressed situations C. Can be given only by way of a peripheral IV line D. Can lead to villous atrophy and cell shrinkage

D. Can lead to villous atrophy and cell shrinkage Disuse of the GI tract has been associated with villous atrophy and generalized cell shrinkage. Translocation of bacteria from the local gut to systemic regions has been noted in relation to GI cell shrinkage, resulting in gram-negative septicemia. PN is administered in a variety of settings, including the patient's home. Patients in highly stressed physiological states such as sepsis, head injury, or burns are candidates for PN therapy. Safe administration of this form of nutrition depends on meticulous management of a central venous catheter.

The patient is elderly and has been diagnosed with Imbalanced nutrition: less than body requirements. Her treatment regimen should include having the nurse: A. Encourage weight gain as rapidly as possible B. Encourage large meals three times a day C. Decrease fluid intake to prevent feeling full D. Encourage fiber intake

D. Encourage fiber intake Increasing fiber intake deters constipation and enhances appetite. Weight gain should be slow and progressive. Frequent small meals should be encouraged to increase dietary intake and to help offset anorexia. Older adults need eight 8-ounce glasses of fluid per day from beverage and food sources.

In measuring the effectiveness of nutritional interventions, the nurse should: A. Expect results to occur rapidly B. Not be concerned with physical measures such as weight C. Expect to maintain a course of action regardless of changes in condition D. Evaluate outcomes according to the patient's expectations and goals

D. Evaluate outcomes according to the patient's expectation and goals The nurse should measure the effectiveness of nutritional interventions by evaluating the patient's expected outcomes and goals of care. Nutrition therapy does not always produce rapid results. Ongoing comparisons need to be made with baseline measures of weight, serum albumin or prealbumin, and protein and kilocalorie intake. Changes in condition may indicate a need to change the nutritional plan of care.

Some proteins are manufactured in the body, but others are not. Those that must be obtained through diet are known as: A. Amino acids B. Dispensable amino acids C. Triglycerides D. Indispensable amino acids

D. Indispensable amino acids The simplest form of protein is the amino acid. The body does not synthesize indispensable amino acids, so these need to be provided in the diet. The body synthesizes dispensable amino acids. Triglycerides are made up of three fatty acids attached to a glycerol.

The patient is admitted with facial trauma, including a broken nose, and has a history of esophageal reflux and of aspiration pneumonia. Given this information, which of the following tubes is appropriate for this patient? A. Nasogastric tube B. Percutaneous endoscopic gastrostomy (PEG) tube C. Nasointestinal tube D. Jejunostomy tube

D. Jejunostomy tube Patients with gastroparesis or esophageal reflux or with a history of aspiration pneumonia may require placement of tubes beyond the stomach into the intestine. The nasogastric tube and the PEG tube are placed in the stomach, and placement could lead to aspiration. The nasointestinal tube and the nasogastric tube may be contraindicated by facial trauma and the broken nose. The jejunostomy tube is the only tube in the list that is beyond the stomach and is not contraindicated by facial trauma.

The patient has a calculated body mass index (BMI) of 34. This would classify the patient as: A. Unclassifiable B. Normal weight C. Overweight D. Obese

D. Obese BMI greater than 30 is defined as obesity. BMI between 25 and 30 is classified as overweight. BMI less than 25 is considered normal or underweight. All patients can be classified by dividing their weight in kilograms by their height in meters squared.

In teaching mothers-to-be about infant nutrition, the nurse instructs patients to: A. Give cow's milk during the first year of life B. Supplement breast milk with corn syrup C. Add honey to infant formulas for increased energy D. Remember that breast milk or formula is sufficient for the first 4 to 6 months

D. Remember that breast milk or formula is sufficient for the first 4 to 6 months Breast milk or formula provides sufficient nutrition for the first 4 to 6 months of life. Infants should not have regular cow's milk during the first year of life. Cow's milk causes gastrointestinal bleeding, is too concentrated for the infant's kidneys to manage, increases the risk of milk product allergies, and is a poor source of iron and vitamins C and E. Honey and corn syrup are potential sources of botulism toxin and should not be used in the infant diet.

The nurse is teaching the patient about dietary guidelines. In discussing the four components of dietary reference intakes (DRIs), it is important to understand that: A. The estimated average requirement (EAR) is appropriate for 100% of the population B. The recommended dietary allowance (RDA) meets the needs of the individual C. Adequate intake (AI) determines the nutrient requirements of the RDA D. The tolerable upper intake level (UL) is not a recommended level of intake

D. The tolerable upper level intake (UL) is not a recommended level of intake The tolerable upper intake level (UL) is the highest level that likely poses no risk of adverse health events. It is not a recommended level of intake. The EAR is the recommended amount of a nutrient that appears sufficient to maintain a specific body function for 50% of the population based on age and gender. The RDA reflects the average needs of 98% of the population, not the exact needs of the individual. AI is the suggested intake for individuals based on observed or experimentally determined estimates of nutrient intakes and is used when evidence is insufficient for setting of the RDA.

The nurse is preparing to insert a nasogastric tube in a patient who is semiconscious. To determine the length of the tube needed to be inserted, the nurse measures from the: A. Tip of the nose to the xiphoid process of the sternum B. Earlobe to the xiphoid process of the sternum C. Tip of the nose to the earlobe D. Tip of the nose to the earlobe to the xiphoid process

D. Tip of the nose to the earlobe to the xiphoid process Measure distance from the tip of the nose to the earlobe to the xiphoid process of the sternum. This approximates the distance from the nose to the stomach in 98% of patients. For duodenal or jejunal placement, an additional 20 to 30 centimeters is required.

The ChooseMyPlate program was developed to replace MyFoodPyramid as a basic guide for buying food and meal preparations. This system was developed by: A. Food and Drug Administration B. 1990 Nutrition Labeling and Education Act C. Referenced daily intakes (RDIs) D. U.S. Department of Agriculture

D. U.S. Department of Agriculture The ChooseMyPlate program was developed by the U.S. Department of Agriculture to replace the MyFoodPyramid program. ChooseMyPlate serves as a basic guide for making food choices for a healthy lifestyle. The Food and Drug Administration (FDA) created daily values for food labels in response to the 1990 Nutrition Labeling and Education Act (NLEA). The FDA first established two sets of reference values: referenced daily intakes (RDIs) and daily reference values (DRVs).

The nurse is instructing a client with hypertension on the importance of choosing foods low in sodium. The nurse should teach the client to limit intake of which food? 1. Apples 2. Bananas 3. Smoked sausage 4. Steamed vegetables

Smoked foods are high in sodium

A client has been diagnosed with iron-deficiency anemia. The doctor has ordered an iron supplement but has also suggested a diet rich in iron. Which of the following foods should be included in the client's discharge iron-rich diet plan? a. Egg yolks b. Bananas c. Cantaloupe d. Peas

a. Egg yolks

The nurse is preparing to teach a client with microcytic hypochromic anemia about the diet to follow after discharge. Which of the following foods should be included in the diet? a. Eggs b. Lettuce c. Citrus fruits d. Cheese

a. Eggs Eggs are high in Iron. Other foods high in iron are organ meats, muscle meats; shellfish, shrimp, and tuna; enriched, whole-grain, and fortified cereals and breads; legumes, nuts, dried fruits, and beans; oatmeal; and sweet potatoes.

Which statements by the mother of a toddler would lead the nurse to suspect that the child has iron-deficiency anemia? Select all that apply. a. "He drinks over 3 cups of milk per day." b. "I can't keep enough apple juice in the house; he must drink over 10 ounces per day." c. "He refuses to eat more than 2 different kinds of vegetables." d. "He doesn't like meat, but he will eat small amounts of it." "He sleeps 12 hours every night and takes 2 naps."

a He drinks over 3 cups of milk per day b. I cant keep enough apple juice in the house; he must drink over 10 oz per day Toddlers should have between 2 and 3 cups of milk per day and 8 ounces of juice per day. If they have more than that, then they are probably not eating enough other foods, including iron-rich foods that have the needed nutrients

When a client is diagnosed with aplastic anemia, the nurse monitors for changes in which of the following physiological functions? a. Bleeding tendencies b. Intake and output c. Peripheral sensation d. Bowel function

a. Bleeding tendencies Aplastic anemia decreases the bone marrow production of RBCs, WBCs, and Platelets. The client is at risk for bruising and bleeding tendencies.

The nurse has provided nutritional teaching on foods high in folate to a client with folate deficiency related to malabsorption syndromes and poor nutrition. Which of the following foods, if chosen by the client, indicates that the client understands the teaching? a. Liver, dark green leafy vegetables b. Whole milk and eggs c. Potatoes and carrots d. Bread and fish

a. Liver, dark green leafy vegetables Foods high in folate are liver, orange juice, cereals, whole grains, beans, nuts, and dark leafy vegetables like spinach

Which of the following symptoms is expected with hemoglobin of 10 g/dl? a. None b. Pallor c. Palpitations d. Shortness of breath

a. None Mild anemia usually has no clinical signs.

From the following teaching tips, choose all that are appropriate for a client with thrombocytopenia. a. Use an electric razor for shaving b. Avoid becoming chilled c. Avoid all skin or body punctures d. Do not scrub skin during bathing e. Eat low-roughage foods f. Avoid use of all aspirin products g. Avoid vigorous blowing of nose h. Use only a soft toothbrush

a. Use an electric razor for shaving c. Avoid all skin or bod punctures e. Eat low-roughage foods f. Avoid use of all aspirin products g. Avoid vigorous blowing of nose h. use only a soft toothbrush

A client was admitted with iron deficiency anemia and blood-streaked emesis. Which question is most appropriate for the nurse to ask in determining the extent of the client's activity intolerance? a. "What activities were you able to do 6 months ago compared with present?" b. "How long have you had this problem?" c. "Have you been able to keep up with all your usual activities?" d. "Are you more tired now than you used to be?"

a. What activities were you able to do 6 months ago compared with present? Rationale: It is difficult to determine activity intolerance without objectively comparing activities from one time frame to another. Because iron deficiency anemia can occur gradually and individual endurance varies, the nurse can best assess the clients activity tolerance by asking the client to compare activities 6 months ago and at the present

A client is beginning a regimen of ferrous sulfate or iron. As you prepare to administer the medication, it is important for you to advise the client that a. Her urine will turn a dark orange b. Her bowel movements will be dark and tarry c. Her appetite will be diminished d. Her vision will become slightly blurred

b. Her bowel movements will be dark and tarry

The usual treatment for iron-deficiency anemia includes: a. Vitamin B12 injection b. Non-enteric-coated ferrous sulfate c. Enteric-coated or sustained-release ferrous sulfate d. Whole blood transfusion

b. Non-enteric-coated ferrous sulfate The usual tx is 325 mg p.o. daily. enteric-coated and sustained rls formulas should be avoided, as they are poorly absorbed

Which of the following foods would the nurse encourage the mother to offer to her child with iron deficiency anemia? a. Rice cereal, whole milk, and yellow vegetables b. Potato, peas, and chicken c. Macaroni, cheese, and ham d. Pudding, green vegetables, and rice

b. Potato, peas, and chicken Potato, peas, chicken, green vegetables, and rice cereal contain significant amounts of iron and therefore would be recommended. Milk and yellow vegetables are not good iron sources. Rice by itself also is not a good source of iron.

When comparing the hematocrit levels of a post-op client, the nurse notes that the hematocrit decreased from 36% to 34% on the third day even though the RBC and hemoglobin values remained stable at 4.5 million and 11.9 g/dL, respectively. Which nursing intervention is most appropriate? a. Check the dressing and drains for frank bleeding b. Call the physician c. Continue to monitor vital signs d. Start oxygen at 2L/min per NC

c. Continue to monitor vital signs The nurse should continue to monitor the client, because this value reflects a normal physiologic response. The physician does not need to be called, and oxygen does not need to be started based on these laboratory findings. Immediately after surgery, the client's hematocrit reflects a falsely high value related to the body's compensatory response to the stress of sudden loss of fluids and blood. Activation of the intrinsic pathway and the renin-angiotensin cycle via antidiuretic hormone produces vasoconstriction and retention of fluid for the first 1 to 2 day post-op. By the second to third day, this response decreases and the client's hematocrit level is more reflective of the amount of RBCs in the plasma. Fresh bleeding is a less likely occurrence on the third post-op day but is not impossible; however, the nurse would have expected to see a decrease in the RBC and hemoglobin values accompanying the hematocrit.

The nurse would instruct the client to eat which of the following foods to obtain the best supply of vitamin B12? a. Whole grains b. Green leafy vegetables c. Meats and dairy products d. Broccoli and Brussel sprouts

c. Meats and dairy products Good sources of vitamin B12 include meats and dairy products. Whole grains are a good source of thiamine. Green leafy vegetables are good sources of niacin, folate, and carotenoids (precursors of vitamin A). Broccoli and Brussels sprouts are good sources of ascorbic acid (vitamin C).

A client with anemia may be tired due to a tissue deficiency of which of the following substances? a. Carbon dioxide b. Factor VIII c. Oxygen d. T-cell antibodies

c. Oxygen Anemia stems from a decreased number of RBCs and the resulting def in O2 and body tiss. Clotting factors, such as 8 relate to the bodies ability to form blood clots and arnt related to anemia, not is carbon dioxide of T antibodies

During physical assessment of a patient, the nurse suspects a chronic, severe iron-deficiency anemia on finding a. Yellow-tinged sclerae b. Gum bleeding and tenderness c. Shiny, smooth tongue d. Numbness of extremities

c. Shiny, smooth tongue Loss of the papillae of the tongue occurs with chronic iron deficiency. Scleral jaundice is associated with hemolysis, gum bleeding and tenderness occur with thrombocytopenia or neutropenia, and extremity numbness is associated with vitamin B12 deficiency or pernicious anemia.

While obtaining a health history from a patient with numerous petechiae on the skin, the nurse asks the patient specifically about the patient's use of a. anticonvulsants. b. oral contraceptives. c. aspirin medications. d. antihypertensives.

c. aspirin meds Salicylates interfere with platelet function and can lead to petechiae and ecchymosis. Anticonvulsants may cause anemia, but not bleeding. Oral contraceptives increase clotting risk. Antihypertensives do not commonly cause problems with decreased clotting.

A patient with a history of iron-deficiency anemia who has not taken iron supplements for several years is experiencing increased fatigue and occasional palpitations. The nurse would expect the patient's laboratory findings to include a. hematocrit (Hct) 38%. b. red blood cell count (RBC) 4,500,000/l. c. hemoglobin (Hgb) 8.6 g/dl (86 g/L). d. normal RBC indices.

c. hgb 8.6 g/dL The patient's clinical manifestations indicate moderate anemia, which is consistent with a Hgb of 6 to 10 g/dl. The other values are all within the range of low-normal to normal.

mother asks the nurse if her child's iron deficiency anemia is related to the child's frequent infections. The nurse responds based on the understanding of which of the following? a. Little is known about iron-deficiency anemia and its relationship to infection in children. b. Children with iron deficiency anemia are more susceptible to infection than are other children. c. Children with iron-deficiency anemia are less susceptible to infection than are other children. d. Children with iron-deficient anemia are equally as susceptible to infection as are other children.

d. Children with iron-deficient anemia are equally as susceptible to infection as are other children Rationale: Children with iron-deficiency anemia are more susceptible to infection because of marked decreases in bone marrow functioning with microcytosis

A client comes into the health clinic 3 years after undergoing a resection of the terminal ileum complaining of weakness, shortness of breath, and a sore tongue. Which client statement indicates a need for intervention and client teaching? a. "I have been drinking plenty of fluids." b. "I have been gargling with warm salt water for my sore tongue." c. "I have 3 to 4 loose stools per day." d. "I take a vitamin B12 tablet every day."

d. I take a vitamin B12 tablet every day Vitamin B12 combines with intrinsic factor in the stomach and is then carried to the ileum, where it is absorbed in the bloodstream. In this situation, vitamin B12 cannot be absorbed regardless of the amount of oral intake of sources of vitamin B12 such as animal protein or vitamin B12 tablets. Vitamin B12 needs to be injected every month, because the ileum has been surgically removed. Replacement of fluids and electrolytes is important when the client has continuous multiple loose stools on a daily basis. Warm salt water is used to soothe sore mucous membranes. Crohn's disease and small bowel resection may cause several loose stools a day.

The physician has ordered several laboratory tests to help diagnose an infant's bleeding disorder. Which of the following tests, if abnormal, would the nurse interpret as most likely to indicate hemophilia? a. Bleeding time b. Tourniquet test c. Clot retraction test d. Partial thromboplastin time (PTT)

d. PTT PTT measures the activity of thromboplastin, which is dependent on intrinsic clotting factors. In hemophilia, the intrinsic clotting factor VIII (antihemophilic factor) is deficient, resulting in a prolonged PTT. Bleeding time reflects platelet function; the tourniquet test measures vasoconstriction and platelet function; and the clot retraction test measures capillary fragility. All of these are unaffected in people with hemophilia.

When caring for a client with a coagulation disorder, your primary focus should be on: a. Prevention of infection b. Pain management c. Reducing edema d. Prevention of injury and hemorrhage

d. Prevention of injury and hemorrhage

A client with iron deficiency anemia is scheduled for discharge. Which instruction about prescribed ferrous gluconate therapy should the nurse include in the teaching plan? a. "Take the medication with an antacid." b. "Take the medication with a glass of milk." c. "Take the medication with cereal." d. "Take the medication on an empty stomach."

d. Take the meds on an empty stomach Preferably, ferrous gluconate should be taken on an empty stomach. Ferrous gluconate should not be taken with antacids, milk, or whole-grain cereals because these foods reduce iron absorption.

A client states that she is afraid of receiving vitamin B12 injections because of the potential toxic reactions. What is the nurse's best response to relieve these fears? a. "Vitamin B12 will cause ringing in the eats before a toxic level is reached." b. "Vitamin B12 may cause a very mild skin rash initially." c. "Vitamin B12 may cause mild nausea but nothing toxic." d. "Vitamin B12 is generally free of toxicity because it is water soluble."

d. Vitamin B12 is generally free of toxicity because it is water soluble Vitamin B12 is a water-soluble vitamin. When water-soluble vitamins are taken in excess of the body's needs, they are filtered through the kidneys and excreted. Vitamin B12 is considered to be nontoxic. Adverse reactions that have occurred are believed to be related to impurities or to the preservative in B12 preparations. Ringing in the ears, skin rash, and nausea are not considered to be related to vitamin B12 administration.

When discussing appropriate food choices with a patient who has iron-deficiency anemia and follows a low-cholesterol diet, the nurse will encourage the patient to increase the dietary intake of a. eggs and muscle meats. b. nuts and cornmeal. c. milk and milk products. d. legumes and dried fruits

d. legumes and dried fruits Rationale: Legumes and dried fruits are high in iron and low in fat and cholesterol.


Ensembles d'études connexes

Anatomy and Physiology Chapters 4-7

View Set

Econ 315 Practice Quiz 4 - CH 10 and 11

View Set